Fundamentals Evolve

A nurse is caring for an older adult who is taking acetaminophen (Tylenol) for the relief of chronic pain. Which substance is most important for the nurse to determine if the client is taking because it intensifies the most serious adverse effect of acetaminophen?
1
Alcohol
2
Caffeine
3
Saw palmetto
4
St. John's wort

Alcohol

The nurse is caring for a client who is on a low carbohydrate diet. With this diet, there is decreased glucose available for energy, and fat is metabolized for energy resulting in an increased production of which substance in the urine?
1
Protein
2
Glucose
3
Ketones
4
Uric acid

Ketones

A nurse manager works on a unit where the nursing staff members are uncomfortable taking care of clients from cultures that are different from their own. How should the nurse manager address this situation?
1
Assign articles about various cultures so that they can become more knowledgeable.
2
Relocate the nurses to units where they will not have to care for clients from a variety of cultures.
3
Rotate the nurses' assignments so they have an equal opportunity to care for clients from other cultures.
4
Plan a workshop that offers opportunities to learn about the cultures they might encounter while at work.

Plan a workshop that offers opportunities to learn about the cultures they might encounter while at work

A client with Addison's disease is receiving cortisone therapy. The nurse expects what clinical indicators if the client abruptly stops the medication? (Select all that apply.)
1
Diplopia
2
Dysphagia
3
Tachypnea
4
Bradycardia
5
Hypotension

Tachypnea Hypotension

A client is admitted to the hospital with a tentative diagnosis of infectious pulmonary tuberculosis. What infection control measures should the nurse take?
1
Don an N95 respirator mask before entering the room.
2
Put on a permeable gown each time before entering the room.
3
Implement contact precautions and post appropriate signage.
4
After finishing with patient care, remove the gown first and then remove the gloves.

Don an N95 respirator mask before entering the room.

To minimize the side effects of the vincristine (Oncovin) that a client is receiving, what type of dietary plan does the nurse expect?
1
Low in fat
2
High in iron
3
High in fluids
4
Low in residue

High in fluids

The client receives a prescription for tap water enemas until clear. The nurse is aware that no more than two enemas should be given at one time to prevent the occurrence of:
1
Hypercalcemia
2
Hypocalcemia
3
Hyperkalemia
4
Hypokalemia

Hypokalemia

A client has been admitted with a urinary tract infection. The nurse receives a urine culture and sensitivity report that reveals the client has Vancomycin Resistant Enterococcus (VRE). After notifying the physician, which action should the nurse take to decrease the risk of transmission to others?
1
Insert a urinary catheter.
2
Initiate Droplet Precautions.
3
Move the client to a private room.
4
Use a high efficiency particulate air (HEPA) respirator during care.?

Move the client to private room

After abdominal surgery a client reports pain. What action should the nurse take first?
1
Reposition the client.
2
Obtain the client's vital signs.
3
Administer the prescribed analgesic.
4
Determine the characteristics of the pain.

Determine the characteristics of the pain

A client is receiving albuterol (Proventil) to relieve severe asthma. For which clinical indicators should the nurse monitor the client? (Select all that apply.)
1
Tremors
2
Lethargy
3
Palpitations
4
Visual disturbances
5
Decreased pulse rate

Tremors and Palpitations

The nurse performs a respiratory assessment and auscultates breath sounds that are high-pitched, creaking and accentuated on expiration. Which term best describes the findings?
1
Rhonchi
2
Wheezes
3
Pleural friction rub
4
Bronchovesicular

Wheezes

When assessing a client's blood pressure, the nurse notes that the blood pressure reading in the right arm is 10 mm Hg higher than the blood pressure reading in
1
is a normal occurrence.
2
may indicate atherosclerosis.
3
can be attributed to aortic disease.
4
indicates lymphedema.

is a normal occurrence.

A client who is to receive radiation therapy for cancer says to the nurse, "My family said I will get a radiation burn." What is the nurse's best response?
1
"Your skin will look like a blistering sunburn."
2
"A localized skin reaction usually occurs."
3
"A daily application of an emollient will prevent a burn."
4
"Your family must have had experience with radiation therapy."

"A localized skin reaction usually occurs.

A 90-year-old female resident of a nursing home falls and fractures the proximal end of her right femur. The surgeon plans to reduce the fracture with an internal fixation device. The general fact about the older adult that the nurse should consider when caring for this client is that:
1
Aging causes a lower pain threshold
2
Physiological coping defenses are reduced
3
Most confused states result from dementia
4
Older adults psychologically tolerate changes well

Physiological coping defenses are reduced

A client is admitted to the hospital because of multiple chronic health problems. What is the priority nursing intervention at this time?

Conducting a multidisciplinary staff conference early during the client's hospitalization

The nurse plans care for a client with a somatoform disorder based on the understanding that the disorder is:
1
A physiological response to stress.
2
A conscious defense against anxiety.
3
An intentional attempt to gain attention.
4
An unconscious means of reducing stress

An unconscious means of reducing stress.

A client has a pressure ulcer that is full thickness with necrosis into the subcutaneous tissue down to the underlying fascia. The nurse should document the assessment finding as which stage of pressure ulcers?

Stage 3

A nurse is caring for a postoperative client who had general anesthesia during surgery. What independent nursing intervention may prevent an accumulation of secretions?
1
Postural drainage
2
Cupping the chest
3
Nasotracheal suctioning
4
Frequent changes of position

Frequent changes of position

A nurse provides discharge teaching related to intermittent urinary self-catheterization to a client with a new spinal cord injury. Which instruction is most important for the nurse to include?
1
"Wear sterile gloves when doing the procedure."
2
"Wash your hands before performing the procedure."
3
"Perform the self-catheterization every 12 hours."
4
"Dispose of the catheter after you have catheterized yourself.

Wash your hands before performing the procedure

A client with hypothermia is brought to the emergency department. What treatment does the nurse anticipate when the patient is in the emergency department?
1
Core rewarming with warm fluids
2
Ambulation to increase metabolism
3
Frequent oral temperature assessments
4
Gastric tube feedings to increase fluid volume

Core rewarming with warm fluids

To prevent footdrop in a client with a leg cast, the nurse should:
1
Encourage complete bed rest to promote healing of the foot.
2
Place the foot in traction.
3
Support the foot with 90 degrees of flexion.
4
Place an elastic stocking on the foot to provide support.

Support the foot with 90 degrees of flexion

A client is receiving albuterol (Proventil) to relieve severe asthma. For which clinical indicators should the nurse monitor the client? (Select all that apply.)
1
Tremors
2
Lethargy
3
Palpitations
4
Visual disturbances
5
Decreased pulse rate

Tremors Palpitations

The nurse performs a respiratory assessment and auscultates breath sounds that are high-pitched, creaking and accentuated on expiration. Which term best describes the findings?
1
Rhonchi
2
Wheezes
3
Pleural friction rub
4
Bronchovesicular

Wheezes

A client has been admitted with a diagnosis of intractable vomiting and can only tolerate sips of water. The initial blood work shows a sodium level of 122 mEq/L and a potassium level of 3.6 mEq/L. Based on the lab results and symptoms, what is the client experiencing?
1
Hypernatremia
2
Hyponatremia
3
Hyperkalemia
4
Hypokalemia

Hyponatremia

A nurse is teaching a community group about the basics of nutrition. A participant questions why fluoride is added to drinking water. The nurse should respond that it is a necessary element added to drinking water to promote:
1
Dental health.
2
Growth and development.
3
Improved hearing.
4
Night vision.

Dental Health

The intake and output of a client over an eight-hour period is: 0800: Intravenous (IV) infusing; 900 mL left in bag; 0830: 150 mL voided; From 0900-1500 time period: 200 mL gastric tube formula + 50 mL water; Repeated x 2.; 1300: 220 mL voided; 1515: 235 mL voided; 1600: IV has 550 mL left in bag. What is the difference between the client's intake and output? Record the answer using a whole number. _________ mL

495 Intake includes 350 mL of IV fluid, 600 mL of nasogastric intubation (NGT) feeding, and 150 mL of water via NGT, for a total intake of 1100 mL; output includes voidings of 150, 220, and 235 mL, for a total output of 605 mL. Subtract 605 mL from 1100 mL for a difference of 495 mL.

A client is scheduled for a transurethral resection of the prostate (TURP). Which statement made by the client most indicates the need for further preoperative teaching?
1
"My urine will be red after surgery."
2
"I will have a catheter after surgery."
3
"My incision will probably be painful."
4
"I will need to drink a lot after surgery."

"My incision will probably be painful.

A client is in a state of uncompensated acidosis. What approximate arterial blood pH does the nurse expect the client to have?
1
7.20
2
7.35
3
7.45
4
7.48

7.20 The pH of blood is maintained within the narrow range of 7.35 to 7.45. When there is an increase in hydrogen ions, the respiratory, buffer, and renal systems attempt to compensate to maintain the pH. If compensation is not successful, acidosis results and is reflected in a lower pH

An adult client presents to the Emergency Department with a nosebleed. After applying pressure, what is the next nursing action?
1
Obtain a medication history from the client
2
Check the blood pressure
3
Instruct the client to avoid picking the nose
4
Check the pulse

Check blood pressure

nurse assesses for hypocalcemia in a postoperative client. One of the initial signs that might be present is:
1
Headache.
2
Pallor.
3
Paresthesias.
4
Blurred vision.

Paresthesias.

A nurse is caring for an elderly client with dementia who has developed dehydration as a result of vomiting and diarrhea. Which assessment best reflects the fluid balance of this client?
1
Skin turgor
2
Intake and output results
3
Client's report about fluid intake
4
Blood lab results

Blood lab results

A client becomes hostile when learning that amputation of a gangrenous toe is being considered. After the client's outburst, what is the best indication that the nurse-client interaction has been therapeutic?

1
Increased physical activity
2
Absence of further outbursts
3
Relaxation of tensed muscles
4
Denial of the need for further discussio

Relaxation of tensed muscles

The client asks the nurse to recommend foods that might be included in a diet for diverticular disease. Which foods would be appropriate to include in the teaching plan? (Select all that apply.)

Whole grains

Cooked fruit and vegetables

Nuts and seeds

Lean red meats

Milk and eggs

Whole grains Cooked fruite and vegetables Milk and eggs

What nursing actions best promote communication when obtaining a nursing history? (Select all that apply.)
1
Establishing eye contact
2
Paraphrasing the client's message
3
Asking "why" and "how" questions
4
Using broad, open-ended statements
5
Reassuring the client that there is no cause for alarm
6
Asking questions that can be answered with a "yes" or "no"

Establishing eye contact Paraphrasing the client's message Using broad, open-ended statements

A dying client is coping with feelings regarding impending death. The nurse bases care on the theory of death and dying by Kübler-Ross. During which stage of grieving should the nurse primarily use nonverbal interventions?
Anger
2
Denial
3
Bargaining
4
Acceptance

Acceptance

An 85-year-old client is alert and able to participate in care. The nurse understands that, according to Erikson, a person's adjustment to the period of senescence will depend largely on adjustment to which developmental stage?
1
Industry versus inferiority
2
Identity versus role confusion
3
Generativity versus stagnation
4
Autonomy versus shame/doubt

Generativity versus stagnation

An 85-year-old client has just been admitted to a nursing home. When designing a plan of care for this older adult the nurse recalls the expected sensory losses associated with aging. (Select all that apply.)

Difficulty in swallowing
2
Diminished sensation of pain
3
Heightened response to stimuli
4
Impaired hearing of high-frequency sounds
5
Increased ability to tolerate environmental heat

Diminished sensation of pain Impaired hearing of high-frequency sounds

A nurse is preparing a community health program for senior citizens. The nurse teaches the group that the physical findings that are typical in older people include:
1
A loss of skin elasticity and a decrease in libido
2
Impaired fat digestion and increased salivary secretions
3
Increased blood pressure and decreased hormone production
4
An increase in body warmth and some swallowing difficulties

Increased blood pressure and decreased hormone production

When a client files a lawsuit against a nurse for malpractice, the client must prove that there is a link between the harm suffered and actions performed by the nurse that were negligent. This is known as:
1
Evidence
2
Tort discovery
3
Proximate cause
4
Common cause

Proximate cause

A client is receiving albuterol (Proventil) to relieve severe asthma. For which clinical indicators should the nurse monitor the client? (Select all that apply.)
1
Tremors
2
Lethargy
3
Palpitations
4
Visual disturbances
5
Decreased pulse rate

Tremors Palpitations

The nurse is providing information about blood pressure to Unlicensed Assistive Personnel (UAP) and recalls that the factor that has the greatest influence on diastolic blood pressure is:
1
Renal function
2
Cardiac output
3
Oxygen saturation
4
Peripheral vascular resistance

Peripheral Vascular resistance

A client comes to the clinic complaining of a productive cough with copious yellow sputum, fever, and chills for the past two days. The first thing the nurse should do when caring for this client is to:
1
Encourage fluids
2
Administer oxygen
3
Take the temperature
4
Collect a sputum specimen

Take the temperature

A nurse speaking in support of the best interest of a vulnerable client reflects the nurse's duty of:
1
Caring.
2
Veracity.
3
Advocacy.
4
Confidentiality.

Advocacy

A physician orders guaifenesin (Humibid) 300 mg four times a day. The dosage strength is 200 milligrams/5 milliliters. To ensure the patient's safety, how many milliliters should the nurse administer for each dose? Record your answer using one decimal place. ____ mL

7.5

The nurse is providing restraint education to a group of nursing students. The nurse should include that it is inappropriate to use a restraint device to:
1
Prevent a client from pulling out an IV when there is concern that the client cannot follow instructions or is confused.
2
Prevent an adult client from getting up at night when there is insufficient staffing on the unit.
3
Maintain immobilization of a client's leg to prevent dislodging a skin graft.
4
Keep an older adult client from falling out of bed following a surgical procedure.

Prevent an adult client from getting up at night when there is insufficient staffing on the unit.

An adolescent is taken to the emergency department of the local hospital after stepping on a nail. The puncture wound is cleansed and a sterile dressing applied. The nurse asks about having had a tetanus immunization. The adolescent responds that all immunizations are up to date. Penicillin is administered, and the client is sent home with instructions to return if there is any change in the wound area. A few days later, the client is admitted to the hospital with a diagnosis of tetanus. Legally, what is the nurse's responsibility in this situation?
1
The nurse's judgment was adequate, and the client was treated accordingly.
2
The possibility of tetanus was not foreseen because the client was immunized.
3
Nurses should routinely administer immunization against tetanus after such an injury.
Correct 4
Data collection by the nurse was incomplete, and as a result the treatment was insufficien

Data collection by the nurse was incomplete, and as a result the treatment was insufficient.

A client experiences a muscle sprain of the ankle. When assessing the injury, the nurse discovers that a hematoma is developing, edema is present and that the client reports tenderness when the ankle is palpated. The nurse anticipates that the plan of care will include the applying of a/an:

Binder
2
Ice bag
3
Elastic bandage
4
Warm compress

Ice bag

A nurse provides crutch-walking instructions to a client that has a left-leg cast. The nurse should explain that weight must be placed:
1
In the axillae.
2
On the hands.
3
On the right side.
4
On the side that the client prefers

On the hands

Which task is most appropriate for a nurse to delegate to unlicensed assistive personnel?
1
Emptying a portable wound drainage device.
2
Instructing a client to use an incentive spirometer.
3
Monitoring the rate of infusion of intravenous fluids.
4
Assessing the blood pressure of a client before physical therapy

Assessing the blood pressure of a client before physical therapy

Nurses are held responsible for the commission of a tort. The nurse understands that a tort is:
1
The application of force to the body of another by a reasonable individual.
2
An illegality committed by one person against the property or person of another.
3
Doing something that a reasonable person under ordinary circumstances would not do.
4
An illegality committed against the public and punishable by the law through the courts

An illegality committed by one person against the property or person of another

Health promotion efforts within the health care system should include efforts related to secondary prevention. Which activities reflect secondary prevention interventions in relation to health promotion? (Select all that apply.)
1
Encouraging regular dental checkups
2
Facilitating smoking cessation programs
3
Administering influenza vaccines to older adults
4
Teaching the procedure for breast self-examination
5
Referring clients with a chronic illness to a support group

Encouraging regular dental checkups Teaching the procedure for breast-self-examination

A client diagnosed with tuberculosis is taking isoniazid (INH). To prevent a food and drug interaction, the nurse should advise the client to avoid:
1
Hot dogs
2
Red wine
3
Sour cream
4
Apple juice

Red wine

When assessing an obese client, a nurse observes dehiscence of the abdominal surgical wound with evisceration. The nurse places the client in the low-Fowler position with the knees slightly bent and encourages the client to lie still. What is the next nursing action?

1
Obtain the vital signs.
2
Notify the health care provider.
3
Reinsert the protruding organs using aseptic technique.
4
Cover the wound with a sterile towel moistened with normal saline.

Cover the wound with a sterile towel moistened with normal saline

The nurse is caring for an older adult client who is aphasic. The client's family reports to the nurse manager that the primary nurse failed to obtain a signed consent form before inserting an indwelling catheter to measure intake and output. What should the nurse manager consider before responding?
1
Procedures for a client's benefit do not require a signed consent.
2
Clients who are aphasic are incapable of signing an informed consent.
3
A separate signed informed consent for routine treatments is unnecessary.
4
A specific intervention without a client's signed consent is an invasion of rights.

A separate signed informed consent for routine treatments is unnecessary

The nurse is preparing discharge instructions for a client that acquired a nosocomial infection, Clostridium difficile. What should the nurse include in the instructions?
1
Anticipate that nausea and vomiting will continue until the infection is no longer present.
2
The infection causes diarrhea accompanied by flatus and abdominal discomfort.
3
Consume a diet that is high in fiber and low in fat.
4
Other than routine handwashing, it is not necessary to perform special disinfection procedures.

The infection causes diarrhea accompanied by flatus and abdominal discomfort

A health care provider prescribes an antibiotic intravenous piggyback (IVPB) twice a day for a client with an infection. The health care provider prescribes peak and trough levels 48 and 72 hours after initiation of the therapy. The client asks the nurse why there is a need for so many blood tests. The nurse's best response is, "These tests will:
1
determine adequate dosage levels of the drug."
2
detect if you are having an allergic reaction to the drug."
3
permit blood culture specimens to be obtained when the drug is at its lowest level."
4
allow comparison of your fever to when the blood level of the antibiotic is at its highest."

Determine adequate dosage levels of the drug

The nurse providing post-procedure care to a client who had a cardiac catheterization through the femoral artery discovers a large amount of blood under the client's buttocks. After donning gloves, which action should the nurse take first?
1
Apply pressure to the site.
2
Obtain vital signs.
3
Change the client's gown and bed linens.
4
Assess the catheterization site.

Assess the catheterization site

When monitoring a client 24 to 48 hours after surgery, the nurse should assess for which problem associated with anesthetic agents?
1
Colitis
2
Stomatitis
3
Paralytic ileus
4
Gastrocolic reflux

Paralytic ileus

A nurse is preparing to administer an oil-retention enema and understands that it works primarily by:
1
Stimulating the urge to defecate.
2
Lubricating the sigmoid colon and rectum.
3
Dissolving the feces.
4
Softening the feces.

Lubricating the sigmoid colon and rectum

A nurse is evaluating the appropriateness of a family member's initial response to grief. What is the most important factor for the nurse to consider?
1
Personality traits
2
Educational level
3
Cultural background
4
Past experiences with death

Cultural background

What should the nurse include in dietary teaching for a client with a colostomy?
Incorrect 1
Liquids should be limited to 1 L per day.
1
Liquids should be limited to 1 L per day.
2
Non-digestible fiber and fruits should be eliminated.
3
A formed stool is an indicator of constipation.
4
The diet should be adjusted to include foods that result in manageable stools

The diet should be adjusted to include foods that result in manageable stools

A client with respiratory difficulties asks why the percussion procedure is being performed. The nurse explains that the primary purpose of percussion is to:
1
Relieve bronchial spasm.
2
Increase depth of respirations.
3
Loosen pulmonary secretions.
4
Expel carbon dioxide from the lungs

Loosen pulmonary secretions

The nurse recognizes that which are important components of a neurovascular assessment? (Select all that apply.)

2
Orientation
3
Capillary refill
4
Pupillary response
5
Respiratory rate
6
Pulse and skin temperature
7
Movement and sensation

capillary refill, pulses, warmth and paresthesias, and movement and sensation

Which age-related change should the nurse consider when formulating a plan of care for an older adult? (Select all that apply.)
1
Difficulty in swallowing
2
Increased sensitivity to heat
3
Increased sensitivity to glare
4
Diminished sensation of pain
5
Heightened response to stimuli

Increased sensitivity to glare Diminished sensation to pain

A client has a right above-the-knee amputation after trauma sustained in a work-related accident. Upon awakening from surgery, the client states, "What happened to me? I don't remember a thing." What is the nurse's best initial response?
1
"Tell me what you think happened."
2
"You will remember more as you get better."
3
"You were in a work-related accident this morning."
4
"It was necessary to amputate your leg after the accident."

You were in a work-related accident this morning

An arterial blood gas report indicates the client's pH is 7.25, PCO2 is 35 mm Hg, and HCO3 is 20 mEq/L. Which disturbance should the nurse identify based on these results?
1
Metabolic acidosis
2
Metabolic alkalosis
3
Respiratory acidosis
4
Respiratory alkalosis

Metabolic acidosis

A client who is to have brain surgery has a signed advance directive in the medical record. In what situation should this document be used?
1
Discharge planning is not covered by insurance.
2
Client cannot consent to his or her own surgery.
3
Postoperative complications occur that require additional treatment.
4
Client death and which client's belongings are to be given to family members

Client cannot consent to his or her own surgery

A nurse is providing colostomy care to a client with a nosocomial infection caused by methicillin-resistant Staphylococcus aureus (MRSA). Which personal protective equipment (PPE) should the nurse use? (Select all that apply.)
1
Gloves
2
Gown
3
Mask
4
Goggles
5
Shoe covers
6
Hair bonnet

Gloves Gown Goggles

What physiological changes that occur with aging must be taken into consideration when the nurse provides care for the older adult? (Select all that apply.)
1
Urinary urgency
2
Loss of skin elasticity
3
Increased body warmth
4
Swallowing difficulties
5
Elevated blood pressure

Urinary urgency Loss of skin elasticity Swallowing difficulties Elevated blood pressure

The nurse recognizes that a common conflict experienced by the older adult is the conflict between:
1
Youth and old age
2
Retirement and work
3
Independence and dependence
4
Wishing to die and wishing to live

Independence and dependence

A client has a paracentesis, and the health care provider removes 1500 mL of fluid. To monitor for a serious postprocedure complication, the nurse should assess for:
1
Dry mouth
2
Tachycardia
3
Hypertensive crisis
4
Increased abdominal distention

Tachycardia

When performing a postoperative assessment, which parameter would alert the nurse to a common side effect of epidural anesthesia?
1
Decreased blood pressure
2
Increased oral temperature
3
Diminished peripheral pulses
4
Unequal bilateral breath sounds

Decreased blood pressure

A nurse is teaching a group of parents about child abuse. What definition of assault should the nurse include in the teaching plan?
1
Assault is a threat to do bodily harm to another person.
2
It is a legal wrong committed by one person against the property of another.
3
It is a legal wrong committed against the public that is punishable by state law.
4
Assault is the application of force to another person without lawful justification.

Assault is a threat to do bodily harm to another person

A client receiving steroid therapy states, "I have difficulty controlling my temper, which is so unlike me, and I don't know why this is happening." What is the nurse's best response?
1
Tell the client it is nothing to worry about.
2
Talk with the client further to identify the specific cause of the problem.
3
Instruct the client to attempt to avoid situations that cause irritation.
4
Interview the client to determine whether other mood swings are being experienced.

Interview the client to determine whether other mood swings are being experienced

A nurse is teaching a group of parents about child abuse. What definition of assault should the nurse include in the teaching plan?
1
Assault is a threat to do bodily harm to another person.
2
It is a legal wrong committed by one person against the property of another.
3
It is a legal wrong committed against the public that is punishable by state law.
4
Assault is the application of force to another person without lawful justification

Assault is a threat to do bodily harm to another person

The intake and output of a client over an eight-hour period is: 0800: Intravenous (IV) infusing; 900 mL left in bag; 0830: 150 mL voided; From 0900-1500 time period: 200 mL gastric tube formula + 50 mL water; Repeated x 2.; 1300: 220 mL voided; 1515: 235 mL voided; 1600: IV has 550 mL left in bag. What is the difference between the client's intake and output? Record the answer using a whole number. _________ mL

495 Intake includes 350 mL of IV fluid, 600 mL of nasogastric intubation (NGT) feeding, and 150 mL of water via NGT, for a total intake of 1100 mL; output includes voidings of 150, 220, and 235 mL, for a total output of 605 mL. Subtract 605 mL from 1100 mL for a difference of 495 mL.

A client with limited mobility is being discharged. To prevent urinary stasis and formation of renal calculi, the nurse should instruct the client to:
1
Increase oral fluid intake to 2 to 3 L per day.
2
Maintain bed rest after discharge.
3
Limit fluid intake to 1 L/day.
4
Void at least every hour

Increase oral fluid intake to 2 to 3 L per day

Immediately after receiving spinal anesthesia a client develops hypotension. To what physiological change does the nurse attribute the decreased blood pressure?
1
Dilation of blood vessels
2
Decreased response of chemoreceptors
3
Decreased strength of cardiac contractions
4
Disruption of cardiac accelerator pathways

Dilation of blood vessels

A nurse is providing preoperative teaching for a client regarding use of an incentive spirometer and should include what instructions?
1
"Inhale completely and exhale in short, rapid breaths."
2
"Inhale deeply through the spirometer, hold it as long as possible, and slowly exhale."
3
"Exhale completely; take a slow, deep breath; hold it as long as possible, and slowly exhale."
4
"Exhale halfway, then inhale a rapid, small breath; repeat several times.

Inhale deeply through the spirometer, hold it as long as possible, and slowly exhale

A client with Type I Diabetes complains of hunger, thirst, tiredness, and frequent urination. Based on these findings, the nurse should take what action?
1
Notify the physician immediately about the client's symptoms.
2
Determine the client's blood glucose level.
3
Administer the client's prescribed insulin.
4
Give the client a peanut butter and graham cracker snack

Determine the client's blood glucose level

What is a nurse's responsibility when administering prescribed opioid analgesics? (Select all that apply.)
1
Count the client's respirations.
2
Document the intensity of the client's pain.
3
Withhold the medication if the client reports pruritus.
4
Verify the number of doses in the locked cabinet before administering the prescribed dose.
5
Discard the medication in the client's toilet before leaving the room if the medication is refused.

Count the client's respirations Document the intensity of the client's pain Verify the number of doses in the locked cabinet before administering the prescribed dose

A client undergoes a bowel resection. When assessing the client 4 hours postoperatively, the nurse identifies which finding as an early sign of shock?
1
Respirations of 10
2
Urine output of 30 ml/hour
3
Lethargy
4
Restlessness

Restlessness

What are the clinical indicators that a nurse expects when an intravenous (IV) line has infiltrated? (Select all that apply.)
1
Heat
2
Pallor
3
Edema
4
Decreased flow rate
5
Increased blood pressure

Pallor Edema Decreased flow rate

The health care provider prescribes a low-fat, 2-gram sodium diet for a client with hypertension. The nurse should explain that the purpose of restricting sodium is to:
1
Chemically stimulate the loop of Henle
2
Diminish the thirst response of the client
3
Prevent reabsorption of water in the distal tubules
4
Cause fluid to move toward the interstitial compartment

Prevent reabsorption of water in the distal tubules

An 89-year-old client with osteoporosis is admitted to the hospital with a compression fracture of the spine. The nurse identifies that a factor of special concern when caring for this client is the client's:
1
Irritability in response to deprivation
2
Decreased ability to recall recent facts
3
Inability to maintain an optimal level of functioning
4
Gradual memory loss resulting from change in environment

Inability to maintain an optimal level of functioning

Health promotion efforts with the chronically ill client should include interventions related to primary prevention. What should this include?
1
Encouraging daily physical exercise
2
Performing yearly physical examinations
3
Providing hypertension screening programs
4
Teaching a person with diabetes how to prevent complications

Encouraging daily physical excercise

Several recently licensed practical nurses are discussing whether they should purchase personal professional liability insurance. Which statement indicates the most accurate information about professional liability insurance?
1
"If you have liability insurance, you are more likely to be sued."
2
"Your employer provides you with the liability insurance you will need."
3
"Liability insurance is not available for nursing professionals working in a hospital."
4
"Personal liability insurance offers representation if the State Board of Nursing files charges against you."

"Personal liability insurance offers representation if the State Board of Nursing files charges against you."

A nurse assesses a client's serum electrolyte levels in the laboratory report. What electrolyte in intracellular fluid should the nurse consider most important?
1
Sodium
2
Calcium
3
Chloride
4
Potassium

Potassium

While caring for a client with a Hemovac portable wound drainage system, the nurse observes that the collection container is half full. The nurse empties the container. What is the next nursing intervention?
1
Encircle the drainage on the dressing.
2
Irrigate the suction tube with sterile saline.
3
Clean the drainage port with an alcohol wipe.
4
Compress the container before closing the port.

Compress the container before closing the port

An 80-year-old female is admitted to the hospital because of complications associated with severe dehydration. The client's daughter asks the nurse how her mother could have become dehydrated because she is alert and able to care for herself. The nurse's best response is:
1
"The body's fluid needs decrease with age because of tissue changes."
2
"Access to fluid may be insufficient to meet the daily needs of the older adult."
3
"Memory declines with age, and the older adult may forget to ingest adequate amounts of fluid."
4
"The thirst reflex diminishes with age, and therefore the recognition of the need for fluid is decreased."

The thirst reflex diminishes with age, and therefore the recognition of the need for fluid is decreased

What is a basic concept associated with rehabilitation that the nurse should consider when formulating discharge plans for clients?
1
Rehabilitation needs are met best by the client's family and community resources.
2
Rehabilitation is a specialty area with unique methods for meeting clients' needs.
3
Immediate or potential rehabilitation needs are exhibited by clients with health problems.
4
Clients who are returning to their usual activities following hospitalization do not require rehabilitation.

Immediate or potential rehabilitation needs are exhibited by clients with health problems.

A nurse educator is presenting information about the nursing process to a class of nursing students. What definition of the nursing process should be included in the presentation?
1
Procedures used to implement client care.
2
Sequence of steps used to meet the client's needs.
3
Activities employed to identify a client's problem.
4
Mechanisms applied to determine nursing goals for the client

Sequence of steps used to meet the client's needs.

Which nursing action is confidential and protected from legal action?
1
Providing health teaching regarding family planning.
2
Offering first aid at the scene of an automobile collision.
3
Reporting incidents of suspected child abuse to the appropriate authorities.
4
Administering resuscitative measures to an unconscious child pulled from a swimming pool.

Reporting incidents of suspected child abuse to the appropriate authorities.

A nurse is evaluating the appropriateness of a family member's initial response to grief. What is the most important factor for the nurse to consider?
1
Personality traits
2
Educational level
3
Cultural background
4
Past experiences with death

Cultural background

What clinical finding indicates to the nurse that a client may have hypokalemia?
1
Edema
2
Muscle spasms
3
Kussmaul breathing
4
Abdominal distention

Abdominal distention

A client is being discharged from the hospital with an indwelling urinary catheter. The client asks about the best way to prevent infection and keep the catheter clean. Which would be appropriate for the nurse to include in the client teaching?
1
Once a day, clean the tubing with a mild soap and water, starting at the drainage bag and moving toward the insertion site.
2
After cleaning the catheter site, it is important to keep the foreskin pushed back for 30 minutes to ensure adequate drying.
3
Clean the insertion site daily using a solution of one part vinegar to two parts water.
4
Change the drainage bag at least once a week as needed.

Change the drainage bag at least once a week as needed

A nurse assesses drainage on a surgical dressing and documents the findings. Which documentation is most informative?
1
"Moderate amount of drainage."
2
"No change in drainage since yesterday."
3"
A 10-mm-diameter area of drainage at 1900 hours."
4
"Drainage is doubled in size since last dressing change."

A 10-mm-diameter area of drainage at 1900 hours."

A nurse is preparing to administer an ophthalmic medication to a client. What techniques should the nurse use for this procedure? (Select all that apply.)
1
Clean the eyelid and eyelashes.
2
Place the dropper against the eyelid.
3
Apply clean gloves before beginning of procedure.
4
Instill the solution directly onto cornea.
5
Press on the nasolacrimal duct after instilling the solution.

Clean the eyelid and eyelashes. Apply clean gloves before beginning of procedure. Press on the nasolacrimal duct after instilling the solution.

A client is being treated for Influenza A (H1N1). The nurse has provided instructions to the client about how to decrease the risk of transmission to others. Which patient statement indicates a need for further instruction/clarification?
1
"I should practice respiratory hygiene/cough etiquette."
2
"I should avoid contact with the elderly or children."
3
"I should obtain a pneumococcal vaccination each year."
4
"I should allow visitors for short periods of time only."

I should obtain a pneumococcal vaccination each year.

After gastric surgery a client has a nasogastric tube in place. What should the nurse do when caring for this client?
1
Monitor for signs of electrolyte imbalance.
2
Change the tube at least once every 48 hours.
3
Connect the nasogastric tube to high continuous suction.
4
Assess placement by injecting 10 mL of water into the tube.

Monitor for signs of electrolyte imbalance.

A nurse is supportive of a child receiving long-term rehabilitation in the home rather than in a health care facility. Why is living with the family so important to a child's emotional development?
1
It provides rewards and punishment.
2
The child's development is supported.
3
It reflects the mores of a larger society.
4
It is where child's identity and roles are learned

It is where child's identity and roles are learned

The nurse is having difficulty understanding a client's decision to have hospice care rather than an extensive surgical procedure. Which ethical principle does the client's behavior illustrate?
1
Justice
2
Veracity
3
Autonomy
4
Beneficence

Autonomy

A client receiving intravenous vancomycin (Vancocin) reports ringing in both ears. Which initial action should the nurse take?
1
Notify the primary healthcare provider.
2
Consult an audiologist.
3
Stop the infusion.
4
Document the finding and continue to monitor the client.

Stop the infusion

A client with a diagnosis of uncontrolled diabetes began receiving Lasix (Furosemide) two days ago. The nurse reviews the morning lab results and discovers that the client's potassium level is 2.8 mEq/L. What is the most appropriate action for the nurse to take?
1
Hold the morning dose of the diuretic and have the lab repeat the test.
2
Continue to monitor the level to ensure that it stays within the normal limits.
3
Notify the primary healthcare provider of the result, which is critically low.
4
Anticipate a prescription for an increase in the dosage of the Lasix.

Notify the primary healthcare provider of the result, which is critically low

The nurse is discussing discharge plans with a client who had a myocardial infarction. The client states, "I'm worried about going home." The nurse responds, "Tell me more about this." What interviewing technique did the nurse use?
1
Exploring
2
Reflecting
3
Refocusing
4
Acknowledging

Exploring

A nurse takes into consideration that the key factor in accurately assessing how a client will cope with body image changes is the:
1
Suddenness of the change
2
Obviousness of the change
3
Extent of the body changes
4
Perception of the body changes

Perception of the body changes

The nurse receives a report on a newly admitted client who is positive for Clostridium difficile. Which category of isolation would the nurse implement for this client?
1
Airborne precautions
2
Droplet precautions
3
Contact precautions
4
Protective environment

Contact precautions

The nurse manager is planning to assign unlicensed assistive personnel (UAP) to care for clients. What care can be delegated on a medical-surgical unit to UAP? (Select all that apply.)
1
Performing a bed bath for a client on bed rest.
2
Evaluating the effectiveness of acetaminophen and codeine (Tylenol #3).
3
Obtaining an apical pulse rate before oral digoxin (Lanoxin) is administered.
4
Assisting a client who has patient-controlled analgesia (PCA) to the bathroom.
5
Assessing the wound integrity of a client recovering from an abdominal laparotomy.

Performing a bed bath for a client on bed rest Assisting a client who has patient-controlled analgesia (PCA) to the bathroom.

When caring for a client with pneumonia, which nursing intervention is the highest priority?
1
Increase fluid intake.
2
Employ breathing exercises and controlled coughing.
3
Ambulate as much as possible.
4
Maintain an NPO status.

Employ breathing exercises and controlled coughing

After changing a dressing that was used to cover a draining wound on a client with Vancomycin Resistant Enterococcus (VRE), the nurse should take which step to ensure proper disposal of soiled dressing?
1
Place the dressing in the bedside trash can.
2
Place the dressing in a red bag/hazardous materials bag.
3
Contact Environmental Services personnel to pick up the dressing.
4
Transport the dressing to the laboratory to be placed in the incinerator.

Place the dressing in a red bag/hazardous materials bag.

The unlicensed assistive person (UAP) assigned to the 7 am shift has not been coming to work until 8 am. Nursing care is delayed and assignments are started late. What is the most appropriate action by the charge nurse/team leader?
1
Discuss the issue with a friend from another unit
2
Remind the UAP of the expected start time
3
Report the problem to the Human Resources department
4
Document the information before discussing it with the UAP

Document the information before discussing it with the UAP

A nurse applies a heating pad to a client's buttocks. Upon removal of the heating pad, the nurse discovers that the client has received burns due to incorrect settings when use of the heating pad was initiated. Which principle would legally apply?
1
No one could be held liable for new equipment.
2
The nurse could be held liable for the injury that occurred.
3
The nurse did what a reasonable, prudent nurse would do.
4
The manufacturer is liable for new equipment.

The nurse could be held liable for the injury that occurred.

The nurse caring for a client with a systemic infection is aware that the assessment finding that is most indicative of a systemic infection is:
1
White blood cell (WBC) count of 8200/mm3
2
Bilateral 3+ pitting pedal edema
3
Oral temperature of 101.3º F
4
Pale skin and nail beds

Oral temperature of 101.3º F

A client with a fractured tibia and fibula is to be discharged from the emergency department with a right leg cast and crutches. In addition to the technical aspects of crutch walking, the nurse should teach the client to:
1
Decrease calcium intake
2
Remove loose rugs from the environment
3
Avoid taking showers until the cast is removed
4
Increase weight bearing on the injured leg gradually

Remove loose rugs from the environment

The nurse is caring for a client admitted with chronic obstructive pulmonary disease (COPD). The nurse should monitor the results of which laboratory test to evaluate the client for hypoxia?
1
Red blood cell count
2
Sputum culture
3
Arterial blood gas
4
Total hemoglobin

Arterial blood gas

The nurse is having difficulty understanding a client's decision to have hospice care rather than an extensive surgical procedure. Which ethical principle does the client's behavior illustrate?
1
Justice
2
Veracity
3
Autonomy
4
Beneficence

Autonomy

A nurse who promotes freedom of choice for clients in decision-making best supports which principle?
1
Justice
2
Autonomy
3
Beneficence
4
Paternalism

Autonomy

A client spends several minutes making negative comments to the nurse about numerous aspects of the hospital stay. What is the nurse's best initial response?
1
Describe the purpose of different hospital therapies to decrease the client's anxiety.
2
Explain that becoming so upset does not allow the client to get much-needed rest.
3
Refocus the conversation on the client's fears, frustrations, and anger about the condition.
4
Permit the client to release feelings and then leave the room to allow the client to regain composure.

Refocus the conversation on the client's fears, frustrations, and anger about the condition.

What should the nurse consider when obtaining an informed consent from a 17-year-old adolescent?
1
If the client is allowed to give consent.
2
The client cannot make informed decisions about health care.
3
If the client is permitted to give voluntary consent when parents are not available.
4
The client probably will be unable to choose between alternatives when asked to consent.

If the client is allowed to give consent.

When changing the soiled bed linens of a client with a wound that is draining seropurulent material, what personal protective equipment (PPE) is most essential for the nurse to wear?
1
Mask
2
Clean gloves
3
Sterile gloves
4
Shoe covers

Sterile gloves

A client with a history of chronic obstructive pulmonary disease (COPD) is admitted with acute bronchopneumonia. The client is in moderate respiratory distress. The nurse should place the client in what position to enhance comfort?
1
Side lying position with head elevated 45 degrees
2
Sim's position with head elevated 90 degrees
3
Semi-Fowler's position with legs elevated
4
High-Fowler's position using the bedside table as an arm rest

High-Fowler's position using the bedside table as an arm rest

A nurse is caring for a client diagnosed with Methicillin-Resistant Staphylococcus Aureus (MRSA) in the urine. The health care provider orders an indwelling urinary catheter to be inserted. Which precaution should the nurse take during this procedure?
1
Droplet precautions
2
Reverse isolation
3
Surgical asepsis
4
Medical asepsis

Surgical asepsis

When changing the soiled bed linens of a client with a wound that is draining seropurulent material, what personal protective equipment (PPE) is most essential for the nurse to wear?
1
Mask
2
Clean gloves
3
Sterile gloves
4
Shoe covers

Clean gloves

The nurse is preparing discharge instructions for a client who was prescribed enalapril maleate (Vasotec) for treatment of hypertension. Which is appropriate for the nurse to include in the client's teaching?
1
Do not change positions suddenly.
2
Light-headedness is a common adverse effect that need not be reported.
3
The medication may cause a sore throat for the first few days.
4
Schedule blood tests weekly for the first 2 months

Do not change positions suddenly

A 50-year-old client being seen for a routine physical asks why a stool specimen for occult blood testing has been prescribed when there is no history of health problems. What is an appropriate nursing response?
1
"You will need to ask your healthcare provider; it is not part of the usual tests for people your age."
2
"There must be concern of a family history of colon cancer; that is a primary reason for an occult blood stool test."
3
"It is performed routinely starting at your age as part of an assessment for colon cancer."
4
"There must have been a positive finding after a digital rectal examination performed by your healthcare provider."

It is performed routinely starting at your age as part of an assessment for colon cancer

An adolescent that had an inguinal hernia repair is being prepared for discharge home. The nurse provides instructions about resumption of physical activities. Which statement by the adolescent indicates that the client understands the instructions?
1
"I can ride my bike in about a week."
2
"I don't have to go to gym class for 3 months."
3
"I can't perform any weightlifting for at least 3 weeks."
4
"I can never participate in football again."

I can't perform any weightlifting for at least 3 weeks

A client has received instructions to take 650 mg aspirin (ASA) every 6 hours as needed for arthritic pain. What should the nurse include in the client's medication teaching? (Select all that apply.)
1
Take the aspirin with meals or a snack.
2
Make an appointment with a dentist if bleeding gums develop.
3
Do not chew enteric-coated tablets.
4
Switch to Tylenol (acetaminophen) if tinnitus occurs.
5
Report persistent abdominal pain.

Take the aspirin with meals or a snack Do not chew enteric-coated tablets Report persistent abdominal pain

A client is admitted to the hospital for an elective surgical procedure. The client tells a nurse about the emotional stress of recently disclosing being a homosexual to family and friends. What is the nurse's first consideration when planning care?
1
Exploring the client's emotional conflict
2
Identifying personal feelings toward this client
3
Planning to discuss this with the client's family
4
Developing a rapport with the client's health care provider

Identifying personal feelings toward this client.

Following a surgery on the neck, the client asks the nurse why the head of the bed is up so high. The nurse should tell the client that the high-Fowler position is preferred for what reason?
1
To avoid strain on the incision
2
To promote drainage of the wound
3
To provide stimulation for the client
4
To reduce edema at the operative site

To reduce edema at the operative site

A client that is scheduled for a surgical resection of the colon and creation of a colostomy for a bowel malignancy asks why preoperative antibiotics have been prescribed. The nurse explains that the primary purpose is to:
1
Decrease peristalsis.
2
Minimize electrolyte imbalance.
3
Decrease bacteria in the intestines.
4
Treat inflammation caused by the malignancy.

Decrease bacteria in the intestinves

The nurse is preparing discharge instructions for a client who has begun to demonstrate signs of early Alzheimer's dementia. The client lives alone. The client's adult children live nearby. According to the prescribed medication regimen the client is to take medications six times throughout the day. What is priority nursing intervention to assist the client with compliance with medication-taking?
1
Contact the client's children and ask them to hire a private duty aide who will provide round-the-clock care.
2
Develop a chart for the client, listing the times the medication should be taken.
3
Contact the primary healthcare provider and discuss the possibility of simplifying the medication regimen.
4
Instruct the client and client's children to put medications in a weekly pill organizer.

Contact the primary healthcare provider and discuss the possibility of simplifying the medication regimen

Which nursing intervention is most appropriate for a client in skeletal traction?
1
Add and remove weights as the client desires.
2
Assess the pin sites at least every shift and as needed.
3
Ensure that the knots in the rope are tied to the pulley.
4
Perform range of motion to joints proximal and distal to the fracture at least once a day.

Assess the pin sites at least every shift and as needed

What is a nurse's responsibility when administering prescribed opioid analgesics? (Select all that apply.)
1
Count the client's respirations.
2
Document the intensity of the client's pain.
3
Withhold the medication if the client reports pruritus.
4
Verify the number of doses in the locked cabinet before administering the prescribed dose.
5
Discard the medication in the client's toilet before leaving the room if the medication is refused.

Count the client's respirations Document the intensity of the client's pain Verify the number of doses in the locked cabinet before administering the prescribed dose

When providing preoperative teaching, the nurse should focus primarily on:
1
Helping the client and family decide if surgery is necessary.
2
Providing emotional support to the client and family.
3
Giving minute-by-minute details of the surgery to the client and family.
4
Providing general information to reduce client and family anxiety.

Providing general information to reduce client and family anxiety

Based on the client's reported pain level, the nurse administers 8 mg of the prescribed morphine. The medication is available in a 10 mg syringe. Wasting of the remaining 2 mg of morphine should be done by the nurse and a witness. It is most appropriate for the nurse to ask which member of the health care team to be the witness?
1
Nursing supervisor
2
Licensed practical nurse (LPN)
3
Client's health care provider
4
Designated nursing assistant

Licensed practical nurse (LPN)

The nurse is caring for a client that is hyperventilating. The nurse recalls that the client is at risk for:
1
Respiratory acidosis
2
Respiratory alkalosis
3
Respiratory compensation
4
Respiratory decompensation

Respiratory alkalosis

When caring for a client with varicella and disseminated herpes zoster, the nurse should implement which types of precautions? (Select all that apply.)
1
Airborne
2
Contact
3
Droplet
4
Hazardous Wastes
5
Standard

Airborne Contact Standard ACS

A client admitted to the hospital with a diagnosis of malabsorption syndrome exhibits signs of tetany. The nurse concludes that the tetany was precipitated by the inadequate absorption of which electrolyte?
1
Sodium
2
Calcium
3
Potassium
4
Phosphorus

Calcium

A client with arthritis increases the dose of ibuprofen (Motrin, Advil) to abate joint discomfort. After several weeks the client becomes increasingly weak. The client is admitted to the hospital and is diagnosed with severe anemia. What clinical indicators does the nurse expect to identify when performing an admission assessment? (Select all that apply.)
1
Melena
2
Tachycardia
3
Constipation
4
Clay-colored stools
5
Painful bowel movements

Melena Tachycardia

A pain scale of 1 to 10 is used by a nurse to assess a client's degree of pain. The client rates the pain as an 8 before receiving an analgesic and a 7 after being medicated. What conclusion should the nurse make regarding the client's response to pain medication?
1
Client has a low pain tolerance.
2
Medication is not adequately effective.
3
Medication has sufficiently decreased the pain level.
4
Client needs more education about the use of the pain scale

Medication is not adequately effective

The nurse has provided instructions about back safety to a client. Which client statement indicates understanding of the instructions?
1
"I should carry objects about 18 inches from my body."
2
"I should sleep on my stomach with a firm mattress."
3
"I should carry objects close to my body."
4
"I should pull rather than push when moving heavy objects."

I should carry objects close to my body

What are the clinical indicators that a nurse expects when an intravenous (IV) line has infiltrated? (Select all that apply.)
1
Heat
2
Pallor
3
Edema
4
Decreased flow rate
5
Increased blood pressur

Pallor Edema Decreased flow rate

A nurse provides discharge teaching related to intermittent urinary self-catheterization to a client with a new spinal cord injury. Which instruction is most important for the nurse to include?
1
"Wear sterile gloves when doing the procedure."
2
"Wash your hands before performing the procedure."
3
"Perform the self-catheterization every 12 hours."
4
"Dispose of the catheter after you have catheterized yourself."

Wash your hands before performing the procedure

To prevent thrombophlebitis in the immediate postoperative period, which action is most important for a nurse to include in the client's plan of care?
1
Increase fluid intake.
2
Restrict fluids.
3
Encourage early mobility.
4
Elevate the knee gatch of the bed.

Encourage early mobility

A nurse is caring for a client with hemiplegia who is frustrated. How can the nurse motivate the client toward independence?
1
Establish long-range goals for the client.
2
Identify errors that the client can correct.
3
Reinforce success in tasks accomplished.
4
Demonstrate ways to promote self-reliance.

Reinforce success in tasks accomplished

The nurse creates a plan of care for a client with a risk of infection. Which is the most desirable expected outcome for the client?
1
All nursing functions will be completed by discharge.
2
All invasive intravenous lines will remain patent.
3
The client will remain awake, alert, and oriented at all times.
4
The client will be free of signs and symptoms of infection by discharge

The client will be free of signs and symptoms of infection by discharge

When assessing an obese client, a nurse observes dehiscence of the abdominal surgical wound with evisceration. The nurse places the client in the low-Fowler position with the knees slightly bent and encourages the client to lie still. What is the next nursing action?
1
Obtain the vital signs.
2
Notify the health care provider.
3
Reinsert the protruding organs using aseptic technique.
4
Cover the wound with a sterile towel moistened with normal saline

Cover the wound with a sterile towel moistened with normal saline

A health care provider prescribes famotidine (Pepcid) and magnesium hydroxide/aluminum hydroxide (Maalox) for a client with a peptic ulcer. The nurse should teach the client to take the Maalox at what time?
1
Only at bedtime, when famotidine is not taken.
2
Only if famotidine is ineffective.
3
At the same time as famotidine, with a full glass or water.
4
One hour before or two hours after famotidine.

One hour before or two hours after famotidine

What is a nurse's responsibility when administering prescribed opioid analgesics? (Select all that apply.)
1
Count the client's respirations.
2
Document the intensity of the client's pain.
3
Withhold the medication if the client reports pruritus.
4
Verify the number of doses in the locked cabinet before administering the prescribed dose.
5
Discard the medication in the client's toilet before leaving the room if the medication is refused.

Count the client's respirations Document the intensity of the client's pain Verify the number of doses in the locked cabinet before administering the prescribed dose

A client's serum potassium level has increased to 5.8 mEq/L. What action should the nurse implement first?
1
Call the laboratory to repeat the test.
2
Take vital signs and notify the charge nurse or health care provider.
3
Inform the cardiac arrest team to place them on alert.
4
Take an electrocardiogram and have lidocaine available

Take vital signs and notify the charge nurse or health care provider

As a nurse prepares an older adult client for sleep, actions are taken to help reduce the likelihood of a fall during the night. What nursing action is most appropriate when targeting older adults' most frequent cause of falls?
1
Moving the client's bedside table closer to the bed.
2
Encouraging the client to take an available sedative.
3
Instructing the client to call the nurse before going to the bathroom.
4
Assisting the client to telephone home to say goodnight to the spouse

Instructing the client to call the nurse before going to the bathroom

A nurse assesses a client with dry and brittle hair, flaky skin, a beefy-red tongue and bleeding gums. The nurse recognizes that these clinical manifestations are most likely a result of:
1
A food allergy.
2
Noncompliance with medications.
3
Side effects from medications.
4
A nutritional deficiency.

A nutritional deficiency

The nurse reviews a medical record and is concerned that the client may develop hyperkalemia. Which disease increases the risk of hyperkalemia?
1
Crohn's
2
Cushing's
3
End-stage renal
4
Gastroesophageal reflux

End-stage renal

A client's intravenous (IV) infusion infiltrates. The nurse concludes that what is most likely the cause of the infiltration?
1
Excessive height of the IV bag
2
Failure to secure the catheter adequately
3
Contamination during the catheter insertion
4
Infusion of a chemically irritating medication

Failure to secure the catheter adequately

The nurse providing post-procedure care to a client who had a cardiac catheterization through the femoral artery discovers a large amount of blood under the client's buttocks. After donning gloves, which action should the nurse take first?
1
Apply pressure to the site.
2
Obtain vital signs.
3
Change the client's gown and bed linens.
4
Assess the catheterization site

Assess the catheterization site

The hospital's policy requires two nurses to supervise the wasting of excess opioid solutions. The nurse draws up the prescribed dose and then requests that another nurse witness wasting of the remaining medication. The second nurse states that there is no time to observe the wasting of the medication, enters the identification to serve as the witness, and leaves the area. What is the appropriate action for the first nurse to take?
1
Waste the appropriate amount of medication and administer the appropriate dose to the client.
2
Accept the second nurse's identification as the witness but ask another nurse to observe the actual wasting.
3
Cancel the process and ask another nurse to serve as the witness and to observe the wasting of the medication.
4
Insist that the second nurse re-enter the area to actually observe the wasting of the medication.

Cancel the process and ask another nurse to serve as the witness and to observe the wasting of the medication.

What does a nurse consider the most significant influence on many clients' perception of pain when interpreting findings from a pain assessment?
1
Age and sex
2
Physical and physiological status
3
Intelligence and economic status
4
Previous experience and cultural values

Previous experience and cultural values

A health care provider prescribes simvastatin (Zocor) 20 mg daily for elevated cholesterol and triglyceride levels for a female client. Which is most important for the nurse to teach when the client initially takes the medication?
1
Take the medication with breakfast.
2
Have liver function tests every six months.
3
Wear sunscreen to prevent photosensitivity reactions.
4
Inform the health care provider if the client wishes to become pregnant.

Inform the health care provider if the client wishes to become pregnant

A client is admitted with metabolic acidosis. The nurse considers that two body systems interact with the bicarbonate buffer system to preserve healthy body fluid pH. What two body systems should the nurse assess for compensatory changes?
1
Skeletal and nervous
2
Circulatory and urinary
3
Respiratory and urinary
4
Muscular and endocrine

Respiratory and urinary

The nurse manager of the unit comes to work obviously intoxicated. The staff nurse's ethical obligation is to:
1
Call the security guard
2
Tell the nurse manager to go home
3
Have the supervisor validate the observation
4
Offer the nurse manager a large cup of coffee

Have the supervisor validate the observation

A nurse is reviewing a plan of care for a client who was admitted with dehydration as a result of prolonged watery diarrhea. Which prescription should the nurse question?
1
Oral psyllium (Metamucil)
2
Oral potassium supplement
3
Parenteral half normal saline
4
Parenteral albumin (Albuminar)

Parenteral albumin (Albuminar)

A nurse is providing colostomy care to a client with a nosocomial infection caused by methicillin-resistant Staphylococcus aureus (MRSA). Which personal protective equipment (PPE) should the nurse use? (Select all that apply.)
1
Gloves
2
Gown
3
Mask
4
Goggles
5
Shoe covers
6
Hair bonnet

Gloves Gown Goggles

A physician orders a urinalysis for a client with an indwelling catheter. To ensure that an appropriate specimen is obtained, the nurse would obtain the specimen from which site?
1
tubing injection port
2
distal end of the tubing
3
urinary drainage bag
4
catheter insertion site

tubing injection port

A nurse reviews a medical record of a client with ascites. What does the nurse identify that may be causing the ascites?
1
Portal hypotension
2
Kidney malfunction
3
Decreased liver function
4
Decreased production of potassium

Decreased liver function

A nurse instructs a client to breathe deeply to open collapsed alveoli. What should the nurse include in the explanation of the relationship between alveoli and improved oxygenation?
1
"The alveoli need oxygen to live."
2
"The alveoli have no direct effect on oxygenation."
3
"Collapsed alveoli increase oxygen demands."
4
"Oxygen is exchanged for carbon dioxide in the alveolar membrane."

Oxygen is exchanged for carbon dioxide in the alveolar membrane

A health care provider prescribes a standard walker (pick-up walker with rubber tips on all four legs). The nurse identifies what clinical findings that indicate the client is capable of using a standard walker?
1
Weak upper arm strength and impaired stamina
2
Weight bearing as tolerated and unilateral paralysis
3
Partial weight bearing on the affected extremity and kyphosis
Correct4
Strong upper arm strength and non-weight bearing on the affected extremity

Strong upper arm strength and non-weight bearing on the affected extermity

A nurse is caring for a client with diarrhea. The nurse anticipates a decrease in which clinical indicator?
1
Pulse rate
2
Tissue turgor
3
Specific gravity
4
Body temperature

Tissue Turgor

A client's chest tube has accidentally dislodged. What is the nursing action of highest priority?
1
Place the client in a left side-lying position.
2
Apply oxygen via non-rebreather mask.
3
Apply a petroleum gauze dressing over the site.
4
Prepare to reinsert a new chest tube

Apply a petroleum gauze dressing over the site

A client has received instructions to take 650 mg aspirin (ASA) every 6 hours as needed for arthritic pain. What should the nurse include in the client's medication teaching? (Select all that apply.)
1
Take the aspirin with meals or a snack.
2
Make an appointment with a dentist if bleeding gums develop.
3
Do not chew enteric-coated tablets.
4
Switch to Tylenol (acetaminophen) if tinnitus occurs.
5
Report persistent abdominal pain.

Take the aspirin with meals or snack Do not chew enteric-coated tablets Report persistent abdominal pain

A client is receiving albuterol (Proventil) to relieve severe asthma. For which clinical indicators should the nurse monitor the client? (Select all that apply.)
1
Tremors
2
Lethargy
3
Palpitations
4
Visual disturbances
5
Decreased pulse rate

Tremors Palpitations

female client explains to the nurse that she sleeps until noon every day and takes frequent naps during the rest of the day. What should the nurse do initially?
Incorrect1
Encourage her to exercise during the day
2
Arrange a referral for a thorough medical evaluation
3
Explain that this behavior is an attempt to avoid facing daily responsibilities
4
Identify that the client is describing clinical findings associated with narcolepsy

Arrange a referral for a thorough medical evaluation

A client is scheduled to receive phenytoin (Dilantin) 100 mg orally at 6 PM but is having difficulty swallowing capsules. What method should the nurse use to help the client take the medication?
1
Sprinkle the powder from the capsule into a cup of water.
2
Insert a rectal suppository containing 100 mg of phenytoin.
3
Contact the prescriber to determine if a change to a suspension form would be possible.
4
Obtain a change in the administration route to allow an intramuscular (IM) injection

Contact the prescriber to determine if a change to a suspension form would be possible

A client with Type I Diabetes complains of hunger, thirst, tiredness, and frequent urination. Based on these findings, the nurse should take what action?
1
Notify the physician immediately about the client's symptoms.
2
Determine the client's blood glucose level.
3
Administer the client's prescribed insulin.
4
Give the client a peanut butter and graham cracker snack

Determine the client's blood glucose level

A nurse is transcribing a practitioner's orders for a group of clients. Which order should the nurse clarify with the practitioner?
1
Discharge in am
2
Blood glucose monitoring ac and bedtime
3
Erythropoietin (Procrit) 6000 units subcutaneously TIW
4
Dalteparin (Fragmin) 5000 international units Sub-Q BID

Erythropoietin (Procrit) 6000 units subcutaneously TIW

The nurse is providing post-procedure care for a client that had a liver biopsy. To prevent hemorrhage, it is the nurse's highest priority to place the client in what position?
1
Prone
2
High-Fowler's
3
On the right side
4
Trendelenburg

On the right side

A client has been admitted with a diagnosis of intractable vomiting and can only tolerate sips of water. The initial blood work shows a sodium level of 122 mEq/L and a potassium level of 3.6 mEq/L. Based on the lab results and symptoms, what is the client experiencing?
1
Hypernatremia
2
Hyponatremia
3
Hyperkalemia
4
Hypokalemia

Hyponatremia

A client is scheduled to receive conscious sedation during a colonoscopy. The client asks the nurse, "How will they 'knock me out' for this procedure?" Which answer by the nurse correctly describes the route of administration for conscious sedation?
1
"You will receive the anesthesia through a face mask."
2
"You will receive medication through an intravenous catheter."
3
"We will give you an oral medication about one hour before the procedure."
4
"The nurse anesthetist will inject the medication into the epidural space of your spine.

You will receive medication through an intravenous catheter

The nurse teaching a health awareness class identifies which situation as being the highest risk factor for the development of a deep vein thrombosis (DVT)?
1
Pregnancy
2
Inactivity
3
Aerobic exercise
4
Tight clothing

Inactivity

A client with an abdominal wound infected with methicillin-resistant Staphylococcus aureus (MRSA) is scheduled for a computed tomography (CT) scan of the abdomen. To ensure client and visitor safety during transport, the nurse should implement which precaution?
1
No special precautions are required.
2
Cover the infected site with a dressing.
3
Drape the client with a covering labeled as biohazardous.
4
Place a surgical mask on the client

Cover the infected site with a dressing

A client is being treated for Influenza A (H1N1). The nurse has provided instructions to the client about how to decrease the risk of transmission to others. Which patient statement indicates a need for further instruction/clarification?
1
"I should practice respiratory hygiene/cough etiquette."
2
"I should avoid contact with the elderly or children."
3
"I should obtain a pneumococcal vaccination each year."
4
"I should allow visitors for short periods of time only."

I should obtain pneumococcal vaccination each year

A client that is scheduled for a surgical resection of the colon and creation of a colostomy for a bowel malignancy asks why preoperative antibiotics have been prescribed. The nurse explains that the primary purpose is to:
1
Decrease peristalsis.
2
Minimize electrolyte imbalance.
3
Decrease bacteria in the intestines.
4
Treat inflammation caused by the malignancy

Decrease bacteria in the intestines

A client with rheumatoid arthritis does not want the prescribed cortisone and informs the nurse. Later, the nurse attempts to administer cortisone. When the client asks what the medication is, the nurse gives an evasive answer. The client takes the medication and later discovers that it was cortisone. The client states an intent to sue. What factors in this situation must be considered in a legal action? (Select all that apply.)
1
Clients have a right to refuse treatment.
2
Nurses are required to answer clients truthfully.
3
The health care provider should have been notified.
4
The client had insufficient knowledge to make such a decision.
5
Legally prescribed medications are administered despite a client's objections.

Clients have a right to refuse treatment Nurses are required to answer clients truthfully The health care provider should have been notified

A client is admitted to the hospital and benazepril hydrochloride (Lotensin) is prescribed for hypertension. Which is an appropriate nursing action for clients taking this medication?
1
Monitor the EEG.
2
Assess for dizziness.
3
Administer the drug after meals.
4
Assess for dark, tarry stools

Assess for dizziness

A health care provider prescribes digoxin (Lanoxin) for a client. The nurse teaches the client to be alert for which common early indication of digoxin toxicity?
1
Nausea
2
Urticaria
3
Photophobia
4
Yellow vision

Nausea

At the conclusion of visiting hours, the parent of a 14-year-old adolescent scheduled for orthopedic surgery the next day hands the nurse a bottle of capsules and says, "These are for my child's allergy. Will you be sure my child takes one about 9 PM tonight?" What is the nurse's best response?
1
"I will give one capsule tonight before bedtime."
2
"I will get a prescription so that the medicine can be taken."
3
"Does your health care provider know about your child's allergy?"
4
"Did you ask your health care provider if your child should have this tonight?

"I will get a prescription so that the medicine can be taken."

A nurse is providing morning hygiene to a bedridden client who was admitted for exacerbation of chronic obstructive pulmonary disease (COPD). What is the priority nursing intervention when the client becomes short of breath during the care?
1
Obtain a pulse oximeter to determine the client's oxygen saturation level.
2
Put the client in a high-Fowler's position.
3
Darken the lights and provide a rest period of at least 15 minutes.
4
Continue the hygiene activities while reassuring the client

Put the client in a high-Fowler's position.

A nurse applies a cold pack to treat an acute musculoskeletal injury. Cold therapy decreases pain by:
1
Promoting analgesia and circulation
2
Numbing the nerves and dilating the blood vessels
3
Promoting circulation and reducing muscle spasms
4
Causing local vasoconstriction, preventing edema and muscle spasm

Causing local vasoconstriction, preventing edema and muscle spasm

When reviewing a drug to be administered, the nurse identifies that the package insert indicates that the Z-track injection technique should be used. Under what circumstance does the nurse expect that this technique will be necessary?
1
Volume of medication to be administered is large.
2
Medication is irritating to subcutaneous tissue and skin.
3
Injection site must be massaged after it is administered.
4
Procedure requires an air bubble to be drawn into the syringe.

Medication is irritating to subcutaneous tissue and skin

When monitoring fluids and electrolytes, the nurse recalls that the major cation-regulating intracellular osmolarity is:
1
Sodium
2
Potassium
3
Calcium
4
Calcitonin

Potassium

What clinical finding does a nurse anticipate when admitting a client with an extracellular fluid volume excess?
1
Rapid, thready pulse
2
Distended jugular veins
3
Elevated hematocrit level
4
Increased serum sodium level

Distended jugular veins

A nurse administers an intravenous solution of 0.45% sodium chloride. In what category of fluids does this solution belong?
1
Isotonic
2
Isomeric
3
Hypotonic
4
Hypertonic

Hypotonic

A nurse who is working on a medical-surgical unit receives a phone call requesting information about a client who has undergone surgery. The nurse observes that the client requested a do not publish ("DNP") order on any information regarding condition or presence in the hospital. What is the best response by the nurse?
1
"We have no record of that client on our unit. Thank you for calling."
2
"The new privacy laws prevent me from providing any client information over the phone."
3
"The client has requested that no information be given out. You'll need to call the client directly."
4
"It is against the hospital's policy to provide you with any information regarding any of our clients."

We have no record of that client on our unit. Thank you for calling." 2

A nurse is assigned to take care of a group of clients. Which client should the nurse see first?
1
A 2-yr-old male with diarrhea
2
A 35-yr-old male who is nauseated
3
A 40-yr-old female who has vomiting due to food poisoning
4
An 83-yr-old female whose last bowel movement was three days ago.

A 2-yr-old male with diarrhea

A nurse is taking care of a client who has severe back pain as a result of a work injury. What nursing considerations should be made when determining the client's plan of care? (Select all that apply.)
1
Ask the client what is the client's acceptable level of pain.
2
Eliminate all activities that precipitate the pain.
3
Administer the pain medications regularly around the clock.
4
Use a different pain scale each time to promote patient education.
5
Assess the client's pain every 15 minutes

Ask the client what is the client's acceptable level of pain. Administer the pain medications regularly around the clock.

What are the desired outcomes that the nurse expects when administering a nonsteroidal antiinflammatory drug (NSAID)? (Select all that apply.)
1
Diuresis
2
Pain relief
3
Antipyresis
4
Bronchodilation
5
Anticoagulation
6
Reduced inflammation

Pain reief Antipyresis Reduced inflammation

A nurse provides teaching for a client who is scheduled for a cholecystectomy. In the initial postoperative period, the nurse explains that the most important part of the treatment plan is:
1
Early ambulation
2
Coughing and deep breathing
3
Wearing anti-embolic elastic stockings
4
Maintenance of a nasogastric tube

Coughing and deep breathing

A primary nurse receives prescriptions for a newly admitted client and has difficulty reading the health care provider's writing. Who should the nurse ask for clarification of this prescription?
1
Nurse practitioner
2
House health care provider that is on-call
3
Health care provider who wrote the prescription
4
Nurse manager familiar with the health care provider's writing

Health care provider who wrote the prescription

A client has a paracentesis, and the health care provider removes 1500 mL of fluid. To monitor for a serious postprocedure complication, the nurse should assess for:
1
Dry mouth
2
Tachycardia
3
Hypertensive crisis
4
Increased abdominal distention

Tachycardia

A nurse is providing morning hygiene to a bedridden client who was admitted for exacerbation of chronic obstructive pulmonary disease (COPD). What is the priority nursing intervention when the client becomes short of breath during the care?
1
Obtain a pulse oximeter to determine the client's oxygen saturation level.
2
Put the client in a high-Fowler's position.
3
Darken the lights and provide a rest period of at least 15 minutes.
4
Continue the hygiene activities while reassuring the client

Put the client in a high-Fowler's position

A client reports severe pain two days after surgery. After assessing the characteristics of the pain, which initial action should the nurse take next?
1
Encourage rest.
2
Obtain the vital signs.
3
Administer the prescribed analgesic.
4
Document the client's pain response.

Obtain the vital signs

What clinical indicators should the nurse expect a client with hyperkalemia to exhibit? (Select all that apply.)
1
Tetany
2
Seizures
3
Diarrhea
4
Weakness
5
Dysrhythmias

Diarrhea Weakness Dysrhythmias

A client is receiving albuterol (Proventil) to relieve severe asthma. For which clinical indicators should the nurse monitor the client? (Select all that apply.)
1
Tremors
2
Lethargy
3
Palpitations
4
Visual disturbances
5
Decreased pulse rate

Tremors Palpitations

A nurse is caring for an elderly client with dementia who has developed dehydration as a result of vomiting and diarrhea. Which assessment best reflects the fluid balance of this client?
1
Skin turgor
2
Intake and output results
3
Client's report about fluid intake
4
Blood lab results

Blood lab results

A client with Addison's disease is receiving cortisone therapy. The nurse expects what clinical indicators if the client abruptly stops the medication? (Select all that apply.)
1
Diplopia
2
Dysphagia
3
Tachypnea
4
Bradycardia
5
Hypotension

Tachypnea Hypotension

What response should a nurse be particularly alert for when assessing a client for side effects of long-term cortisone therapy?
1
Hypoglycemia
2
Severe anorexia
3
Anaphylactic shock
4
Behavioral changes

Behavioral changes

A client with arthritis increases the dose of ibuprofen (Motrin, Advil) to abate joint discomfort. After several weeks the client becomes increasingly weak. The client is admitted to the hospital and is diagnosed with severe anemia. What clinical indicators does the nurse expect to identify when performing an admission assessment? (Select all that apply.)
1
Melena
2
Tachycardia
3
Constipation
4
Clay-colored stools
5
Painful bowel movements

Melena Tachycardia

Neomycin, 1 gram, is prescribed preoperatively for a client with cancer of the colon. The client asks why this is necessary. How should the nurse respond?
1
"It is used to prevent you from getting a bladder infection before surgery."
2
"It will decrease your kidney function and lessen urine production during surgery."
3
"It will kill the bacteria in your bowel and decrease the risk for infection after surgery."
4
"It is used to alter the body flora, which reduces spread of the tumor to adjacent organs."

"It will kill the bacteria in your bowel and decrease the risk for infection after surgery."

A nurse is assisting a client to transfer from the bed to a chair. What should the nurse do to widen the client's base of support during the transfer?
1
Spread the client's feet away from each other.
2
Move the client on the count of three.
3
Instruct the client to flex the muscles of the internal girdle.
4
Stand close to the client when assisting with the move

Spread the client's feet away from each other.

A client has Clostridium difficile. The nurse is providing discharge instructions related to decreasing the risk of transmission to family members. What would be appropriate to include in the client's teaching?
1
Increase fluids.
2
Increase fiber in the diet.
3
Wash hands with soap and water.
4
Wash hands with alcohol based hand sanitizer

Wash hands with soap and water.

The nurse assesses an elderly client with a diagnosis of dehydration and recognizes which finding as an early sign of dehydration?
1
Sunken eyes
2
Dry, flaky skin
3
Change in mental status
4
Decreased bowel sounds

Change in mental status

he nurse administers a pneumococcal vaccine to a 70-year-old client. The client asks "Will I have to get this every year like I do with the flu shot?" How should the nurse respond?
1
"You need to receive the pneumococcal vaccine every other year."
2
"The pneumococcal vaccine should be received in early autumn every year."
3
"You should get the flu and pneumococcal vaccines at your annual physical examination."
Correct 4
"It is unnecessary to have any follow-up injections of the pneumococcal vaccine after this dose.

It is unnecessary to have any follow-up injections of the pneumococcal vaccine after this dose

A nurse assesses for hypocalcemia in a postoperative client. One of the initial signs that might be present is:
1
Headache.
2
Pallor.
3
Paresthesias.
4
Blurred vision

Parasthesias

The nurse is caring for a client who is receiving therapy for vitamin B12 deficiency. Which finding indicates that the therapy is having the desired effect?
1
Normal serum electrolyte levels
2
Healthy skin integrity
3
Resolution of peripheral edema
4
Improved hemoglobin and hematocrit levels

Improved hemoglobin and hematocrit levels

What clinical finding does a nurse anticipate when admitting a client with an extracellular fluid volume excess?
1
Rapid, thready pulse
2
Distended jugular veins
3
Elevated hematocrit level
4
Increased serum sodium level

Distended jugular veins

A visitor from a room adjacent to a client asks the nurse what disease the client has. The nurse responds, "I cannot discuss any client's illness with you." What legal issue supports the nurse's response?
1
Libel
2
Slander
3
Negligence
4
Invasion of privacy

Invasion of privacy

A client has been admitted with a urinary tract infection. The nurse receives a urine culture and sensitivity report that reveals the client has Vancomycin Resistant Enterococcus (VRE). After notifying the physician, which action should the nurse take to decrease the risk of transmission to others?
1
Insert a urinary catheter.
2
Initiate Droplet Precautions.
3
Move the client to a private room.
4
Use a high efficiency particulate air (HEPA) respirator during care

Move the client to a private room

The nurse is monitoring a client's hemoglobin level. The nurse recalls that the amount of hemoglobin in the blood has what effect on oxygenation status?
1
Except with rare blood disorders, hemoglobin seldom affects oxygenation status.
2
There are many other factors that impact oxygenation status more than hemoglobin does.
3
A low hemoglobin level causes reduced oxygen-carrying capacity.
4
Hemoglobin reflects the body's clotting ability and may or may not impact oxygenation status.

A low hemoglobin level causes reduced oxygen-carrying capacity

A nurse addresses the needs of a client who is hyperventilating to prevent what complication?
1
Cardiac arrest
2
Carbonic acid deficit
3
Reduction in serum pH
4
Excess oxygen saturation

Carbonic acide deficit

A client with an abdominal wound infected with methicillin-resistant Staphylococcus aureus (MRSA) is scheduled for a computed tomography (CT) scan of the abdomen. To ensure client and visitor safety during transport, the nurse should implement which precaution?
1
No special precautions are required.
2
Cover the infected site with a dressing.
3
Drape the client with a covering labeled as biohazardous.
4
Place a surgical mask on the client.

Cover the infected site with a dressing

A nurse manager is evaluating the performance of the LPN/LVN who is supervising Unlicensed Assistive Personnel (UAP). What action indicates to the nurse manager that the LPN/LVN needs further instruction?
1
Requests that the UAP take vital signs on the clients assigned to their team.
2
Asks the UAP to assess the client's response to a respiratory treatment
3
Instructs the UAP to communicate to a client that the meal trays will be delayed.
4
Collaborates with the UAP to determine the best time to ambulate a client.

Ask the UAP to assess the client's response to a respiratory treatment

A nurse is taking care of a client who has severe back pain as a result of a work injury. What nursing considerations should be made when determining the client's plan of care? (Select all that apply.)
1
Ask the client what is the client's acceptable level of pain.
2
Eliminate all activities that precipitate the pain.
3
Administer the pain medications regularly around the clock.
4
Use a different pain scale each time to promote patient education.
5
Assess the client's pain every 15 minutes

Ask the client what is the client's acceptable level of pain. Administer the pain medications regularly around the clock.

When caring for a client with pneumonia, which nursing intervention is the highest priority?
1
Increase fluid intake.
2
Employ breathing exercises and controlled coughing.
3
Ambulate as much as possible.
4
Maintain an NPO status

Employ breathing exercises and controlled coughing.

A nurse is caring for a client for whom segmental postural drainage treatments are prescribed. The nurse should avoid scheduling the treatment at what time?
1
At bedtime
2
After a meal
3
One hour before a meal
4
One hour after awakening

After a meal

A child is being treated with oral ampicillin (Omnipen) for otitis media. What should be included in the discharge instructions that the nurse provides to the parents of the client?
1
Complete the entire course of antibiotic therapy.
2
Herbal fever remedies are highly discouraged.
3
Administer the medication with meals.
4
Stop the antibiotic therapy when the child no longer has a fever

Complete the entire course of antibiotic therapy.

A health care provider prescribes transdermal fentanyl (Duragesic) 25 mcg/hr every 72 hours. During the first 24 hours after starting the fentanyl, what is the most important nursing intervention?
1
Change the dose until pain is tolerable.
2
Manage pain with oral pain medication.
3
Assess the client for anticholinergic side effects.
4
Instruct the client to take the medication with food

Manage pain with oral pain medication

When suctioning a client with a tracheostomy, an important safety measure for the nurse is to:
1
Hyperventilate the client with room air prior to suctioning.
2
Apply suction only as the catheter is being withdrawn.
3
Insert the catheter until the cough reflex is stimulated.
4
Remove the inner cannula before inserting the suction catheter.

Apply suction only as the catheter is being withdrawn

A client has undergone a subtotal thyroidectomy. The client is being transferred from the post anesthesia care unit/recovery area to the inpatient nursing unit. What emergency equipment is most important for the nurse to have available for this client?
1
A defibrillator
2
An IV infusion pump
3
A tracheostomy tray
4
An electrocardiogram (ECG) monito

A tracheostomy tray

A nurse is obtaining a health history from the newly admitted client who has chronic pain in the knee. What should the nurse include in the pain assessment? (Select all that apply.)
1
Pain history including location, intensity and quality of pain
2
Client's purposeful body movement in arranging the papers on the bedside table
3
Pain pattern including precipitating and alleviating factors
4
Vital signs such as increased blood pressure and heart rate
5
The client's family statement about increases in pain with ambulation

Pain history including location, intensity and quality of pain Pain pattern including precipitating and alleviating factors

A client who sustained a large open wound as a result of an accident is receiving daily sterile dressing changes. To maintain sterility when changing the dressing, the nurse should:
1
Put the unopened sterile glove package carefully on the sterile field
2
Remove the sterile drape from its package by lifting it by the corners
3
Don sterile gloves before opening the package containing the field drape
4
Pour irrigation liquid from a height of at least three inches above the sterile containe

Remove the sterile drape from its package by lifting it by the corners.

The nurse assesses a client's pulse and documents the strength of the pulse as 3+. The nurse understands that this indicates the pulse is:
1
diminished.
2
normal.
3
full.
4
bounding

full

A nurse manager is evaluating the performance of the LPN/LVN who is supervising Unlicensed Assistive Personnel (UAP). What action indicates to the nurse manager that the LPN/LVN needs further instruction?
1
Requests that the UAP take vital signs on the clients assigned to their team.
Correct 2
Asks the UAP to assess the client's response to a respiratory treatment
3
Instructs the UAP to communicate to a client that the meal trays will be delayed.
4
Collaborates with the UAP to determine the best time to ambulate a client.

Asks the UAP to assess the client's response to a respiratory treatment

A nurse is reviewing a client's plan of care. What is the determining factor in the revision of the plan?
1
Time available for care
2
Validity of the problem
3
Method for providing care
4
Effectiveness of the interventions

Effectiveness of the interventions

A nurse has provided discharge instructions to a client who received a prescription for a walker to use for assistance with ambulation. The nurse determines that the teaching has been effective when the client:
1
Picks up the walker and carries it for short distances.
2
Uses the walker only when someone else is present.
3
Moves the walker no more than 12 inches in front of the client during use.
4
States that a walker will be purchased on the way home from the hospital.

Moves the walker no more than 12 inches in front of the client during use

A client with a terminal illness reaches the stage of acceptance. How can the nurse best help the client during this stage?
1
Acknowledge the client's crying.
2
Encourage unrestricted family visits.
3
Explain details of the care being given.
4
Stay nearby without initiating conversation

Stay nearby without initiating conversation

A nurse is obtaining a health history from the newly admitted client who has chronic pain in the knee. What should the nurse include in the pain assessment? (Select all that apply.)
1
Pain history including location, intensity and quality of pain
2
Client's purposeful body movement in arranging the papers on the bedside table
3
Pain pattern including precipitating and alleviating factors
4
Vital signs such as increased blood pressure and heart rate
5
The client's family statement about increases in pain with ambulation

Pain history including location, intensity and quality of pain Pain pattern including precipitating and alleviating factors

The nurse should monitor for which involuntary physiological response in a client who is experiencing pain?
1
Crying
2
Splinting
3
Perspiring
4
Grimacing

Perspiring

What are the best ways for a nurse to be protected legally? (Select all that apply.)
1
Ensure that a therapeutic relationship with all clients has been established.
2
Provide care within the parameters of the state's nurse practice act.
3
Carry at least $100,000 worth of liability insurance.
4
Document consistently and objectively.
5
Clearly document a client's non-adherence to the medical regimen

Provide care within the parameters of the state's nurse practice act. Document consistently and objectively. Clearly document a client's non-adherence to the medical regimen

Which drug requires the nurse to monitor the client for signs of hyperkalemia?
1
Furosemide (Lasix)
2
Metolazone (Zaroxolyn)
3
Spironolactone (Aldactone)
4
Hydrochlorothiazide (HydroDIURIL)

Spironolactone

An 85-year-old client is alert and able to participate in care. The nurse understands that, according to Erikson, a person's adjustment to the period of senescence will depend largely on adjustment to which developmental stage?
1
Industry versus inferiority
2
Identity versus role confusion
3
Generativity versus stagnation
4
Autonomy versus shame/doubt

Generativity versus role stagnation

A client is admitted to the hospital for an elective surgical procedure. The client tells a nurse about the emotional stress of recently disclosing being a homosexual to family and friends. What is the nurse's first consideration when planning care?
1
Exploring the client's emotional conflict
2
Identifying personal feelings toward this client
3
Planning to discuss this with the client's family
4
Developing a rapport with the client's health care provide

Identifying personal feelings toward this client

A dying client is coping with feelings regarding impending death. The nurse bases care on the theory of death and dying by Kübler-Ross. During which stage of grieving should the nurse primarily use nonverbal interventions?
1
Anger
2
Denial
3
Bargaining
4
Acceptance

Acceptance

A nurse applies an ice pack to a client's leg for 20 minutes. The cold application will cause what physiological effect?
1
Local anesthesia
2
Peripheral vasodilation
3
Depression of vital signs
4
Decreased viscosity of blood

Local Anasthesia

While caring for a client with a Hemovac portable wound drainage system, the nurse observes that the collection container is half full. The nurse empties the container. What is the next nursing intervention?
1
Encircle the drainage on the dressing.
2
Irrigate the suction tube with sterile saline.
3
Clean the drainage port with an alcohol wipe.
4
Compress the container before closing the port

Compress the container before closing the port

When suctioning a client with a tracheostomy, an important safety measure for the nurse is to:
1
Hyperventilate the client with room air prior to suctioning.
2
Apply suction only as the catheter is being withdrawn.
3
Insert the catheter until the cough reflex is stimulated.
4
Remove the inner cannula before inserting the suction catheter.

Apply suction only as the catheter is being withdrawn.

What effect of povidone-iodine (Betadine) does a nurse consider when using it on the client's skin before obtaining a specimen for a blood culture?
1
Makes the skin more supple
2
Avoids drying the skin as does alcohol
3
Eliminates surface bacteria that may contaminate the culture
4
Provides a cooling agent to diminish the feeling from the puncture wound

Eliminates surface bacteria that may contaminate the culture

A nurse is caring for a client who has a Hemovac portable wound suction device after abdominal surgery. What is the reason why the nurse empties the device when it is half full?
1
Emptying the unit is safer when it is half full.
2
Accurate measurement of drainage is facilitated.
3
Negative pressure in the unit lessens as fluid accumulates, interfering with further drainage.
4
Fluid collecting in the unit exerts positive pressure, forcing drainage back up the tubing and into the wound

Negative pressure in the unit lessens as fluid accumulates, interfering with further drainage.

The nurse instructs a client that, in addition to building bones and teeth, calcium is also important for:
1
Bile production.
2
Blood production.
3
Blood clotting.
4
Digestion of fats

Blood clotting

When meeting the unique preoperative teaching needs of an older adult, the nurse plans a teaching program based on the principle that learning:
1
Reduces general anxiety
2
Is negatively affected by aging
3
Requires continued reinforcement
4
Necessitates readiness of the learner

Requires continued reinforcement

A nurse is helping a client who observes the traditional Jewish dietary laws to prepare a dietary menu. What considerations should the nurse make?
1
Eating beef and veal is prohibited.
2
Consumption of fish with scales is forbidden.
3
Meat and milk at the same meal are forbidden.
4
Consuming alcohol, coffee, and tea are prohibited

Meat and milk at the same meal are forbidden

A client being treated for Influenza A (H1N1) is scheduled for a computed tomography (CT) scan. To ensure client and visitor safety during transport, the nurse should take which precaution?
1
Place a surgical mask on the client.
2
Other than Standard Precautions, no additional precautions are needed.
3
Minimize close physical contact.
4
Cover the client's legs with a blanke

Place a surgical mask on the client

A nurse is reviewing a client's plan of care. What is the determining factor in the revision of the plan?
1
Time available for care
2
Validity of the problem
3
Method for providing care
4
Effectiveness of the interventions

Effectiveness of the interventions

A nurse is evaluating the effectiveness of treatment for a client with excessive fluid volume. What clinical finding indicates that treatment has been successful?
1
Clear breath sounds
2
Positive pedal pulses
3
Normal potassium level
4
Increased urine specific gravity

Clear breath sounds

When providing care for a client with a nasogastric (NG) tube, the nurse should take measures to prevent what serious complication?
1
Skin breakdown
2
Aspiration pneumonia
3
Retention ileus
4
Profuse diarrhea

Aspiration pneumonia

A client who experienced extensive burns is receiving IV fluids to replace fluid loss. The nurse should monitor for which initial sign of fluid overload?
1
Crackles in the lungs
2
Decreased heart rate
3
Decreased blood pressure
4
Cyanosis

Crackles in the lungs

A client with a history of ulcerative colitis is admitted to the hospital because of severe rectal bleeding. The client engages in angry outbursts and places excessive demands on the staff. One day an unlicensed assistive personnel (UAP) tells the nurse, "I've had it. I am not putting up with that behavior. I'm not going in there again." What is the best response by the nurse?
1
"You need to try to be patient. The client is going through a lot right now."
2
"I'll talk with the client. Maybe I can figure out the best way for us to handle this."
3
"Just ignore it and get on with your work. I'll assign someone else to take a turn."
4
"The client's frightened and taking it out on the staff. Let's think of approaches we can take."

"The client's frightened and taking it out on the staff. Let's think of approaches we can take."

When nurses are conducting health assessment interviews with older clients, they should:
1
Leave a written questionnaire for clients to complete at their leisure
2
Ask family members rather than the client to supply the necessary information
3
Spend time in several short sessions to elicit more complete information from the clients
4
Keep referring to previous questions to ascertain that the information given by clients is correct

Spend time in several short sessions to elicit more complete information from the clients

A nursing supervisor sends unlicensed assistive personnel (UAP) to help relieve the burden of care on a short-staffed medical-surgical unit. Which tasks can be delegated to UAP? (Select all that apply.)
1
Taking routine vital signs.
2
Applying a sterile dressing.
3
Answering clients' call lights.
4
Administering saline infusions.
5
Changing linens on an occupied bed.
6
Assessing client responses to ambulation.

Taking routine vital signs. Answering clients' call lights. Changing linens on an occupied bed.

The nurse is interviewing a client admitted for uncontrolled diabetes after binging on alcohol for the past two weeks. The client states "I am worried about how I am going to pay my bills for my family while I am hospitalized." Which statement by the nurse would best elicit information from the client?
1
"You are worried about paying your bills?"
2
"Don't worry; your bills will get paid eventually."
3
"When was the last time you were admitted for hyperglycemia?"
4
"You really shouldn't be drinking alcohol because of your diagnosis of diabetes"

"You are worried about paying your bills?"

A nurse is caring for a newly admitted client in a long-term care facility. The nurse notes that the client has a decreased attention span and cannot concentrate. The nurse suspects which effects of sensory deprivation?
1
Cognitive response
2
Emotional response
3
Perceptual response
4
Physical response

Cognitive response

A nurse receives a shift report on four adult clients that are between the ages of 25-55. Which client should the nurse assess first?
1
Male client with a hemoglobin of 15.9
2
Female client on warfarin (Coumadin) with an International Normalized Ratio (INR) of 7.5
3
Female client taking daily calcium supplements with a serum calcium level of 9.4
4
Male client with a blood urea nitrogen (BUN) of 20 and a creatinine of 1.1

Female client on warfarin (Coumadin) with an International Normalized Ratio (INR) of 7.5

A nurse is discussing weight loss with an obese individual with Ménière's disease. Which suggestion by the nurse is most important?
1
Limit intake to 900 calories a day.
2
Enroll in an exercise class.
3
Get involved in diversionary activities when there is an urge to eat.
4
Keep a diary of all foods eaten each day.

Keep a diary of all foods eaten each day

A client reports nausea, vomiting, and seeing a yellow light around objects. A diagnosis of hypokalemia is made. Upon a review of the client's prescribed medication list, the nurse determines that what is the likely cause of the clinical findings?
1
Digoxin (Lanoxin)
2
Furosemide (Lasix)
3
Propranolol (Inderal)
4
Spironolactone (Aldactone

Digoxin

The nurse is caring for a client that underwent a rhinoplasty surgical procedure 5 hours ago. After administering pain medication, the nurse notes the client is swallowing frequently. The nurse understands that the cause of frequent swallowing is most likely caused from:
1
A normal response to the analgesic
2
Oral dryness caused by nasal packing
3
An adverse reaction to anesthesia
4
Bleeding posterior to the nasal packing

Bleeding posterior to the nasal packing

A client has seeds containing radium implanted in the pharyngeal area. What should the nurse include in the client's plan of care?
1
Have the client void every two hours.
2
Maintain the client in an isolation room.
3
Spend time with the client to allow verbalization of feelings.
4
Wear two pairs of gloves when touching the client during care.

Maintain the client in an isolation room

A health care provider has prescribed isoniazid (Laniazid) for a client. Which instruction should the nurse give the client about this medication?
1
Prolonged use can cause dark concentrated urine.
2
The medication is best absorbed when taken on an empty stomach.
3
Take the medication with aluminum hydroxide to minimize GI upset.
4
Drinking alcohol daily can cause drug-induced hepatitis.

Drinking alcohol daily can cause drug-induced hepatitis

Which nursing interventions require a nurse to wear gloves? (Select all that apply.)
1
Giving a back rub
2
Cleaning a newborn immediately after delivery
3
Emptying a portable wound drainage
4
Interviewing a client in the emergency department
5
Obtaining the blood pressure of a client who is human immunodeficiency virus (HIV) positive.

Cleaning a newborn immediately after delivery Emptying a portable wound drainage

An older client who is receiving chemotherapy for cancer has severe nausea and vomiting and becomes dehydrated. The client is admitted to the hospital for rehydration therapy. Which interventions have specific gerontologic implications the nurse must consider? (Select all that apply.)
1
Assessment of skin turgor
2
Documentation of vital signs
3
Assessment of intake and output
4
Administration of antiemetic drugs
5
Replacement of fluid and electrolytes

Assessment of skin turgor Administration of antiemetic drugs Replacement of fluid and electrolytes

Health promotion efforts within the health care system should include efforts related to secondary prevention. Which activities reflect secondary prevention interventions in relation to health promotion? (Select all that apply.)
1
Encouraging regular dental checkups
2
Facilitating smoking cessation programs
3
Administering influenza vaccines to older adults
4
Teaching the procedure for breast self-examination
5
Referring clients with a chronic illness to a support group

Encouraging regular dental checkups Teaching the procedure for breast self-examination

To decrease abdominal distention following a client's surgery, what actions should the nurse take? (Select all that apply.)
1
Encourage ambulation
2
Give sips of ginger ale
3
Auscultate bowel sounds
4
Provide a straw for drinking
5
Offer an opioid analgesic

Encourage ambulation Auscultate bowel sounds

A client becomes anxious after being scheduled for a colostomy. What is the most effective way for the nurse to help the client?
1
Administer the prescribed as needed (prn) sedative.
2
Encourage the client to express feelings.
3
Explain the postprocedure course of treatment.
4
Reassure the client that there are others with this problem

Encourage the client to express feelings.

Fundamentals Evolve - Subjecto.com

Fundamentals Evolve

Your page rank:

Total word count: 16605
Pages: 60

Calculate the Price

- -
275 words
Looking for Expert Opinion?
Let us have a look at your work and suggest how to improve it!
Get a Consultant

A nurse is caring for an older adult who is taking acetaminophen (Tylenol) for the relief of chronic pain. Which substance is most important for the nurse to determine if the client is taking because it intensifies the most serious adverse effect of acetaminophen?
1
Alcohol
2
Caffeine
3
Saw palmetto
4
St. John’s wort

Alcohol

The nurse is caring for a client who is on a low carbohydrate diet. With this diet, there is decreased glucose available for energy, and fat is metabolized for energy resulting in an increased production of which substance in the urine?
1
Protein
2
Glucose
3
Ketones
4
Uric acid

Ketones

A nurse manager works on a unit where the nursing staff members are uncomfortable taking care of clients from cultures that are different from their own. How should the nurse manager address this situation?
1
Assign articles about various cultures so that they can become more knowledgeable.
2
Relocate the nurses to units where they will not have to care for clients from a variety of cultures.
3
Rotate the nurses’ assignments so they have an equal opportunity to care for clients from other cultures.
4
Plan a workshop that offers opportunities to learn about the cultures they might encounter while at work.

Plan a workshop that offers opportunities to learn about the cultures they might encounter while at work

A client with Addison’s disease is receiving cortisone therapy. The nurse expects what clinical indicators if the client abruptly stops the medication? (Select all that apply.)
1
Diplopia
2
Dysphagia
3
Tachypnea
4
Bradycardia
5
Hypotension

Tachypnea Hypotension

A client is admitted to the hospital with a tentative diagnosis of infectious pulmonary tuberculosis. What infection control measures should the nurse take?
1
Don an N95 respirator mask before entering the room.
2
Put on a permeable gown each time before entering the room.
3
Implement contact precautions and post appropriate signage.
4
After finishing with patient care, remove the gown first and then remove the gloves.

Don an N95 respirator mask before entering the room.

To minimize the side effects of the vincristine (Oncovin) that a client is receiving, what type of dietary plan does the nurse expect?
1
Low in fat
2
High in iron
3
High in fluids
4
Low in residue

High in fluids

The client receives a prescription for tap water enemas until clear. The nurse is aware that no more than two enemas should be given at one time to prevent the occurrence of:
1
Hypercalcemia
2
Hypocalcemia
3
Hyperkalemia
4
Hypokalemia

Hypokalemia

A client has been admitted with a urinary tract infection. The nurse receives a urine culture and sensitivity report that reveals the client has Vancomycin Resistant Enterococcus (VRE). After notifying the physician, which action should the nurse take to decrease the risk of transmission to others?
1
Insert a urinary catheter.
2
Initiate Droplet Precautions.
3
Move the client to a private room.
4
Use a high efficiency particulate air (HEPA) respirator during care.?

Move the client to private room

After abdominal surgery a client reports pain. What action should the nurse take first?
1
Reposition the client.
2
Obtain the client’s vital signs.
3
Administer the prescribed analgesic.
4
Determine the characteristics of the pain.

Determine the characteristics of the pain

A client is receiving albuterol (Proventil) to relieve severe asthma. For which clinical indicators should the nurse monitor the client? (Select all that apply.)
1
Tremors
2
Lethargy
3
Palpitations
4
Visual disturbances
5
Decreased pulse rate

Tremors and Palpitations

The nurse performs a respiratory assessment and auscultates breath sounds that are high-pitched, creaking and accentuated on expiration. Which term best describes the findings?
1
Rhonchi
2
Wheezes
3
Pleural friction rub
4
Bronchovesicular

Wheezes

When assessing a client’s blood pressure, the nurse notes that the blood pressure reading in the right arm is 10 mm Hg higher than the blood pressure reading in
1
is a normal occurrence.
2
may indicate atherosclerosis.
3
can be attributed to aortic disease.
4
indicates lymphedema.

is a normal occurrence.

A client who is to receive radiation therapy for cancer says to the nurse, "My family said I will get a radiation burn." What is the nurse’s best response?
1
"Your skin will look like a blistering sunburn."
2
"A localized skin reaction usually occurs."
3
"A daily application of an emollient will prevent a burn."
4
"Your family must have had experience with radiation therapy."

"A localized skin reaction usually occurs.

A 90-year-old female resident of a nursing home falls and fractures the proximal end of her right femur. The surgeon plans to reduce the fracture with an internal fixation device. The general fact about the older adult that the nurse should consider when caring for this client is that:
1
Aging causes a lower pain threshold
2
Physiological coping defenses are reduced
3
Most confused states result from dementia
4
Older adults psychologically tolerate changes well

Physiological coping defenses are reduced

A client is admitted to the hospital because of multiple chronic health problems. What is the priority nursing intervention at this time?

Conducting a multidisciplinary staff conference early during the client’s hospitalization

The nurse plans care for a client with a somatoform disorder based on the understanding that the disorder is:
1
A physiological response to stress.
2
A conscious defense against anxiety.
3
An intentional attempt to gain attention.
4
An unconscious means of reducing stress

An unconscious means of reducing stress.

A client has a pressure ulcer that is full thickness with necrosis into the subcutaneous tissue down to the underlying fascia. The nurse should document the assessment finding as which stage of pressure ulcers?

Stage 3

A nurse is caring for a postoperative client who had general anesthesia during surgery. What independent nursing intervention may prevent an accumulation of secretions?
1
Postural drainage
2
Cupping the chest
3
Nasotracheal suctioning
4
Frequent changes of position

Frequent changes of position

A nurse provides discharge teaching related to intermittent urinary self-catheterization to a client with a new spinal cord injury. Which instruction is most important for the nurse to include?
1
"Wear sterile gloves when doing the procedure."
2
"Wash your hands before performing the procedure."
3
"Perform the self-catheterization every 12 hours."
4
"Dispose of the catheter after you have catheterized yourself.

Wash your hands before performing the procedure

A client with hypothermia is brought to the emergency department. What treatment does the nurse anticipate when the patient is in the emergency department?
1
Core rewarming with warm fluids
2
Ambulation to increase metabolism
3
Frequent oral temperature assessments
4
Gastric tube feedings to increase fluid volume

Core rewarming with warm fluids

To prevent footdrop in a client with a leg cast, the nurse should:
1
Encourage complete bed rest to promote healing of the foot.
2
Place the foot in traction.
3
Support the foot with 90 degrees of flexion.
4
Place an elastic stocking on the foot to provide support.

Support the foot with 90 degrees of flexion

A client is receiving albuterol (Proventil) to relieve severe asthma. For which clinical indicators should the nurse monitor the client? (Select all that apply.)
1
Tremors
2
Lethargy
3
Palpitations
4
Visual disturbances
5
Decreased pulse rate

Tremors Palpitations

The nurse performs a respiratory assessment and auscultates breath sounds that are high-pitched, creaking and accentuated on expiration. Which term best describes the findings?
1
Rhonchi
2
Wheezes
3
Pleural friction rub
4
Bronchovesicular

Wheezes

A client has been admitted with a diagnosis of intractable vomiting and can only tolerate sips of water. The initial blood work shows a sodium level of 122 mEq/L and a potassium level of 3.6 mEq/L. Based on the lab results and symptoms, what is the client experiencing?
1
Hypernatremia
2
Hyponatremia
3
Hyperkalemia
4
Hypokalemia

Hyponatremia

A nurse is teaching a community group about the basics of nutrition. A participant questions why fluoride is added to drinking water. The nurse should respond that it is a necessary element added to drinking water to promote:
1
Dental health.
2
Growth and development.
3
Improved hearing.
4
Night vision.

Dental Health

The intake and output of a client over an eight-hour period is: 0800: Intravenous (IV) infusing; 900 mL left in bag; 0830: 150 mL voided; From 0900-1500 time period: 200 mL gastric tube formula + 50 mL water; Repeated x 2.; 1300: 220 mL voided; 1515: 235 mL voided; 1600: IV has 550 mL left in bag. What is the difference between the client’s intake and output? Record the answer using a whole number. _________ mL

495 Intake includes 350 mL of IV fluid, 600 mL of nasogastric intubation (NGT) feeding, and 150 mL of water via NGT, for a total intake of 1100 mL; output includes voidings of 150, 220, and 235 mL, for a total output of 605 mL. Subtract 605 mL from 1100 mL for a difference of 495 mL.

A client is scheduled for a transurethral resection of the prostate (TURP). Which statement made by the client most indicates the need for further preoperative teaching?
1
"My urine will be red after surgery."
2
"I will have a catheter after surgery."
3
"My incision will probably be painful."
4
"I will need to drink a lot after surgery."

"My incision will probably be painful.

A client is in a state of uncompensated acidosis. What approximate arterial blood pH does the nurse expect the client to have?
1
7.20
2
7.35
3
7.45
4
7.48

7.20 The pH of blood is maintained within the narrow range of 7.35 to 7.45. When there is an increase in hydrogen ions, the respiratory, buffer, and renal systems attempt to compensate to maintain the pH. If compensation is not successful, acidosis results and is reflected in a lower pH

An adult client presents to the Emergency Department with a nosebleed. After applying pressure, what is the next nursing action?
1
Obtain a medication history from the client
2
Check the blood pressure
3
Instruct the client to avoid picking the nose
4
Check the pulse

Check blood pressure

nurse assesses for hypocalcemia in a postoperative client. One of the initial signs that might be present is:
1
Headache.
2
Pallor.
3
Paresthesias.
4
Blurred vision.

Paresthesias.

A nurse is caring for an elderly client with dementia who has developed dehydration as a result of vomiting and diarrhea. Which assessment best reflects the fluid balance of this client?
1
Skin turgor
2
Intake and output results
3
Client’s report about fluid intake
4
Blood lab results

Blood lab results

A client becomes hostile when learning that amputation of a gangrenous toe is being considered. After the client’s outburst, what is the best indication that the nurse-client interaction has been therapeutic?

1
Increased physical activity
2
Absence of further outbursts
3
Relaxation of tensed muscles
4
Denial of the need for further discussio

Relaxation of tensed muscles

The client asks the nurse to recommend foods that might be included in a diet for diverticular disease. Which foods would be appropriate to include in the teaching plan? (Select all that apply.)

Whole grains

Cooked fruit and vegetables

Nuts and seeds

Lean red meats

Milk and eggs

Whole grains Cooked fruite and vegetables Milk and eggs

What nursing actions best promote communication when obtaining a nursing history? (Select all that apply.)
1
Establishing eye contact
2
Paraphrasing the client’s message
3
Asking "why" and "how" questions
4
Using broad, open-ended statements
5
Reassuring the client that there is no cause for alarm
6
Asking questions that can be answered with a "yes" or "no"

Establishing eye contact Paraphrasing the client’s message Using broad, open-ended statements

A dying client is coping with feelings regarding impending death. The nurse bases care on the theory of death and dying by Kübler-Ross. During which stage of grieving should the nurse primarily use nonverbal interventions?
Anger
2
Denial
3
Bargaining
4
Acceptance

Acceptance

An 85-year-old client is alert and able to participate in care. The nurse understands that, according to Erikson, a person’s adjustment to the period of senescence will depend largely on adjustment to which developmental stage?
1
Industry versus inferiority
2
Identity versus role confusion
3
Generativity versus stagnation
4
Autonomy versus shame/doubt

Generativity versus stagnation

An 85-year-old client has just been admitted to a nursing home. When designing a plan of care for this older adult the nurse recalls the expected sensory losses associated with aging. (Select all that apply.)

Difficulty in swallowing
2
Diminished sensation of pain
3
Heightened response to stimuli
4
Impaired hearing of high-frequency sounds
5
Increased ability to tolerate environmental heat

Diminished sensation of pain Impaired hearing of high-frequency sounds

A nurse is preparing a community health program for senior citizens. The nurse teaches the group that the physical findings that are typical in older people include:
1
A loss of skin elasticity and a decrease in libido
2
Impaired fat digestion and increased salivary secretions
3
Increased blood pressure and decreased hormone production
4
An increase in body warmth and some swallowing difficulties

Increased blood pressure and decreased hormone production

When a client files a lawsuit against a nurse for malpractice, the client must prove that there is a link between the harm suffered and actions performed by the nurse that were negligent. This is known as:
1
Evidence
2
Tort discovery
3
Proximate cause
4
Common cause

Proximate cause

A client is receiving albuterol (Proventil) to relieve severe asthma. For which clinical indicators should the nurse monitor the client? (Select all that apply.)
1
Tremors
2
Lethargy
3
Palpitations
4
Visual disturbances
5
Decreased pulse rate

Tremors Palpitations

The nurse is providing information about blood pressure to Unlicensed Assistive Personnel (UAP) and recalls that the factor that has the greatest influence on diastolic blood pressure is:
1
Renal function
2
Cardiac output
3
Oxygen saturation
4
Peripheral vascular resistance

Peripheral Vascular resistance

A client comes to the clinic complaining of a productive cough with copious yellow sputum, fever, and chills for the past two days. The first thing the nurse should do when caring for this client is to:
1
Encourage fluids
2
Administer oxygen
3
Take the temperature
4
Collect a sputum specimen

Take the temperature

A nurse speaking in support of the best interest of a vulnerable client reflects the nurse’s duty of:
1
Caring.
2
Veracity.
3
Advocacy.
4
Confidentiality.

Advocacy

A physician orders guaifenesin (Humibid) 300 mg four times a day. The dosage strength is 200 milligrams/5 milliliters. To ensure the patient’s safety, how many milliliters should the nurse administer for each dose? Record your answer using one decimal place. ____ mL

7.5

The nurse is providing restraint education to a group of nursing students. The nurse should include that it is inappropriate to use a restraint device to:
1
Prevent a client from pulling out an IV when there is concern that the client cannot follow instructions or is confused.
2
Prevent an adult client from getting up at night when there is insufficient staffing on the unit.
3
Maintain immobilization of a client’s leg to prevent dislodging a skin graft.
4
Keep an older adult client from falling out of bed following a surgical procedure.

Prevent an adult client from getting up at night when there is insufficient staffing on the unit.

An adolescent is taken to the emergency department of the local hospital after stepping on a nail. The puncture wound is cleansed and a sterile dressing applied. The nurse asks about having had a tetanus immunization. The adolescent responds that all immunizations are up to date. Penicillin is administered, and the client is sent home with instructions to return if there is any change in the wound area. A few days later, the client is admitted to the hospital with a diagnosis of tetanus. Legally, what is the nurse’s responsibility in this situation?
1
The nurse’s judgment was adequate, and the client was treated accordingly.
2
The possibility of tetanus was not foreseen because the client was immunized.
3
Nurses should routinely administer immunization against tetanus after such an injury.
Correct 4
Data collection by the nurse was incomplete, and as a result the treatment was insufficien

Data collection by the nurse was incomplete, and as a result the treatment was insufficient.

A client experiences a muscle sprain of the ankle. When assessing the injury, the nurse discovers that a hematoma is developing, edema is present and that the client reports tenderness when the ankle is palpated. The nurse anticipates that the plan of care will include the applying of a/an:

Binder
2
Ice bag
3
Elastic bandage
4
Warm compress

Ice bag

A nurse provides crutch-walking instructions to a client that has a left-leg cast. The nurse should explain that weight must be placed:
1
In the axillae.
2
On the hands.
3
On the right side.
4
On the side that the client prefers

On the hands

Which task is most appropriate for a nurse to delegate to unlicensed assistive personnel?
1
Emptying a portable wound drainage device.
2
Instructing a client to use an incentive spirometer.
3
Monitoring the rate of infusion of intravenous fluids.
4
Assessing the blood pressure of a client before physical therapy

Assessing the blood pressure of a client before physical therapy

Nurses are held responsible for the commission of a tort. The nurse understands that a tort is:
1
The application of force to the body of another by a reasonable individual.
2
An illegality committed by one person against the property or person of another.
3
Doing something that a reasonable person under ordinary circumstances would not do.
4
An illegality committed against the public and punishable by the law through the courts

An illegality committed by one person against the property or person of another

Health promotion efforts within the health care system should include efforts related to secondary prevention. Which activities reflect secondary prevention interventions in relation to health promotion? (Select all that apply.)
1
Encouraging regular dental checkups
2
Facilitating smoking cessation programs
3
Administering influenza vaccines to older adults
4
Teaching the procedure for breast self-examination
5
Referring clients with a chronic illness to a support group

Encouraging regular dental checkups Teaching the procedure for breast-self-examination

A client diagnosed with tuberculosis is taking isoniazid (INH). To prevent a food and drug interaction, the nurse should advise the client to avoid:
1
Hot dogs
2
Red wine
3
Sour cream
4
Apple juice

Red wine

When assessing an obese client, a nurse observes dehiscence of the abdominal surgical wound with evisceration. The nurse places the client in the low-Fowler position with the knees slightly bent and encourages the client to lie still. What is the next nursing action?

1
Obtain the vital signs.
2
Notify the health care provider.
3
Reinsert the protruding organs using aseptic technique.
4
Cover the wound with a sterile towel moistened with normal saline.

Cover the wound with a sterile towel moistened with normal saline

The nurse is caring for an older adult client who is aphasic. The client’s family reports to the nurse manager that the primary nurse failed to obtain a signed consent form before inserting an indwelling catheter to measure intake and output. What should the nurse manager consider before responding?
1
Procedures for a client’s benefit do not require a signed consent.
2
Clients who are aphasic are incapable of signing an informed consent.
3
A separate signed informed consent for routine treatments is unnecessary.
4
A specific intervention without a client’s signed consent is an invasion of rights.

A separate signed informed consent for routine treatments is unnecessary

The nurse is preparing discharge instructions for a client that acquired a nosocomial infection, Clostridium difficile. What should the nurse include in the instructions?
1
Anticipate that nausea and vomiting will continue until the infection is no longer present.
2
The infection causes diarrhea accompanied by flatus and abdominal discomfort.
3
Consume a diet that is high in fiber and low in fat.
4
Other than routine handwashing, it is not necessary to perform special disinfection procedures.

The infection causes diarrhea accompanied by flatus and abdominal discomfort

A health care provider prescribes an antibiotic intravenous piggyback (IVPB) twice a day for a client with an infection. The health care provider prescribes peak and trough levels 48 and 72 hours after initiation of the therapy. The client asks the nurse why there is a need for so many blood tests. The nurse’s best response is, "These tests will:
1
determine adequate dosage levels of the drug."
2
detect if you are having an allergic reaction to the drug."
3
permit blood culture specimens to be obtained when the drug is at its lowest level."
4
allow comparison of your fever to when the blood level of the antibiotic is at its highest."

Determine adequate dosage levels of the drug

The nurse providing post-procedure care to a client who had a cardiac catheterization through the femoral artery discovers a large amount of blood under the client’s buttocks. After donning gloves, which action should the nurse take first?
1
Apply pressure to the site.
2
Obtain vital signs.
3
Change the client’s gown and bed linens.
4
Assess the catheterization site.

Assess the catheterization site

When monitoring a client 24 to 48 hours after surgery, the nurse should assess for which problem associated with anesthetic agents?
1
Colitis
2
Stomatitis
3
Paralytic ileus
4
Gastrocolic reflux

Paralytic ileus

A nurse is preparing to administer an oil-retention enema and understands that it works primarily by:
1
Stimulating the urge to defecate.
2
Lubricating the sigmoid colon and rectum.
3
Dissolving the feces.
4
Softening the feces.

Lubricating the sigmoid colon and rectum

A nurse is evaluating the appropriateness of a family member’s initial response to grief. What is the most important factor for the nurse to consider?
1
Personality traits
2
Educational level
3
Cultural background
4
Past experiences with death

Cultural background

What should the nurse include in dietary teaching for a client with a colostomy?
Incorrect 1
Liquids should be limited to 1 L per day.
1
Liquids should be limited to 1 L per day.
2
Non-digestible fiber and fruits should be eliminated.
3
A formed stool is an indicator of constipation.
4
The diet should be adjusted to include foods that result in manageable stools

The diet should be adjusted to include foods that result in manageable stools

A client with respiratory difficulties asks why the percussion procedure is being performed. The nurse explains that the primary purpose of percussion is to:
1
Relieve bronchial spasm.
2
Increase depth of respirations.
3
Loosen pulmonary secretions.
4
Expel carbon dioxide from the lungs

Loosen pulmonary secretions

The nurse recognizes that which are important components of a neurovascular assessment? (Select all that apply.)

2
Orientation
3
Capillary refill
4
Pupillary response
5
Respiratory rate
6
Pulse and skin temperature
7
Movement and sensation

capillary refill, pulses, warmth and paresthesias, and movement and sensation

Which age-related change should the nurse consider when formulating a plan of care for an older adult? (Select all that apply.)
1
Difficulty in swallowing
2
Increased sensitivity to heat
3
Increased sensitivity to glare
4
Diminished sensation of pain
5
Heightened response to stimuli

Increased sensitivity to glare Diminished sensation to pain

A client has a right above-the-knee amputation after trauma sustained in a work-related accident. Upon awakening from surgery, the client states, "What happened to me? I don’t remember a thing." What is the nurse’s best initial response?
1
"Tell me what you think happened."
2
"You will remember more as you get better."
3
"You were in a work-related accident this morning."
4
"It was necessary to amputate your leg after the accident."

You were in a work-related accident this morning

An arterial blood gas report indicates the client’s pH is 7.25, PCO2 is 35 mm Hg, and HCO3 is 20 mEq/L. Which disturbance should the nurse identify based on these results?
1
Metabolic acidosis
2
Metabolic alkalosis
3
Respiratory acidosis
4
Respiratory alkalosis

Metabolic acidosis

A client who is to have brain surgery has a signed advance directive in the medical record. In what situation should this document be used?
1
Discharge planning is not covered by insurance.
2
Client cannot consent to his or her own surgery.
3
Postoperative complications occur that require additional treatment.
4
Client death and which client’s belongings are to be given to family members

Client cannot consent to his or her own surgery

A nurse is providing colostomy care to a client with a nosocomial infection caused by methicillin-resistant Staphylococcus aureus (MRSA). Which personal protective equipment (PPE) should the nurse use? (Select all that apply.)
1
Gloves
2
Gown
3
Mask
4
Goggles
5
Shoe covers
6
Hair bonnet

Gloves Gown Goggles

What physiological changes that occur with aging must be taken into consideration when the nurse provides care for the older adult? (Select all that apply.)
1
Urinary urgency
2
Loss of skin elasticity
3
Increased body warmth
4
Swallowing difficulties
5
Elevated blood pressure

Urinary urgency Loss of skin elasticity Swallowing difficulties Elevated blood pressure

The nurse recognizes that a common conflict experienced by the older adult is the conflict between:
1
Youth and old age
2
Retirement and work
3
Independence and dependence
4
Wishing to die and wishing to live

Independence and dependence

A client has a paracentesis, and the health care provider removes 1500 mL of fluid. To monitor for a serious postprocedure complication, the nurse should assess for:
1
Dry mouth
2
Tachycardia
3
Hypertensive crisis
4
Increased abdominal distention

Tachycardia

When performing a postoperative assessment, which parameter would alert the nurse to a common side effect of epidural anesthesia?
1
Decreased blood pressure
2
Increased oral temperature
3
Diminished peripheral pulses
4
Unequal bilateral breath sounds

Decreased blood pressure

A nurse is teaching a group of parents about child abuse. What definition of assault should the nurse include in the teaching plan?
1
Assault is a threat to do bodily harm to another person.
2
It is a legal wrong committed by one person against the property of another.
3
It is a legal wrong committed against the public that is punishable by state law.
4
Assault is the application of force to another person without lawful justification.

Assault is a threat to do bodily harm to another person

A client receiving steroid therapy states, "I have difficulty controlling my temper, which is so unlike me, and I don’t know why this is happening." What is the nurse’s best response?
1
Tell the client it is nothing to worry about.
2
Talk with the client further to identify the specific cause of the problem.
3
Instruct the client to attempt to avoid situations that cause irritation.
4
Interview the client to determine whether other mood swings are being experienced.

Interview the client to determine whether other mood swings are being experienced

A nurse is teaching a group of parents about child abuse. What definition of assault should the nurse include in the teaching plan?
1
Assault is a threat to do bodily harm to another person.
2
It is a legal wrong committed by one person against the property of another.
3
It is a legal wrong committed against the public that is punishable by state law.
4
Assault is the application of force to another person without lawful justification

Assault is a threat to do bodily harm to another person

The intake and output of a client over an eight-hour period is: 0800: Intravenous (IV) infusing; 900 mL left in bag; 0830: 150 mL voided; From 0900-1500 time period: 200 mL gastric tube formula + 50 mL water; Repeated x 2.; 1300: 220 mL voided; 1515: 235 mL voided; 1600: IV has 550 mL left in bag. What is the difference between the client’s intake and output? Record the answer using a whole number. _________ mL

495 Intake includes 350 mL of IV fluid, 600 mL of nasogastric intubation (NGT) feeding, and 150 mL of water via NGT, for a total intake of 1100 mL; output includes voidings of 150, 220, and 235 mL, for a total output of 605 mL. Subtract 605 mL from 1100 mL for a difference of 495 mL.

A client with limited mobility is being discharged. To prevent urinary stasis and formation of renal calculi, the nurse should instruct the client to:
1
Increase oral fluid intake to 2 to 3 L per day.
2
Maintain bed rest after discharge.
3
Limit fluid intake to 1 L/day.
4
Void at least every hour

Increase oral fluid intake to 2 to 3 L per day

Immediately after receiving spinal anesthesia a client develops hypotension. To what physiological change does the nurse attribute the decreased blood pressure?
1
Dilation of blood vessels
2
Decreased response of chemoreceptors
3
Decreased strength of cardiac contractions
4
Disruption of cardiac accelerator pathways

Dilation of blood vessels

A nurse is providing preoperative teaching for a client regarding use of an incentive spirometer and should include what instructions?
1
"Inhale completely and exhale in short, rapid breaths."
2
"Inhale deeply through the spirometer, hold it as long as possible, and slowly exhale."
3
"Exhale completely; take a slow, deep breath; hold it as long as possible, and slowly exhale."
4
"Exhale halfway, then inhale a rapid, small breath; repeat several times.

Inhale deeply through the spirometer, hold it as long as possible, and slowly exhale

A client with Type I Diabetes complains of hunger, thirst, tiredness, and frequent urination. Based on these findings, the nurse should take what action?
1
Notify the physician immediately about the client’s symptoms.
2
Determine the client’s blood glucose level.
3
Administer the client’s prescribed insulin.
4
Give the client a peanut butter and graham cracker snack

Determine the client’s blood glucose level

What is a nurse’s responsibility when administering prescribed opioid analgesics? (Select all that apply.)
1
Count the client’s respirations.
2
Document the intensity of the client’s pain.
3
Withhold the medication if the client reports pruritus.
4
Verify the number of doses in the locked cabinet before administering the prescribed dose.
5
Discard the medication in the client’s toilet before leaving the room if the medication is refused.

Count the client’s respirations Document the intensity of the client’s pain Verify the number of doses in the locked cabinet before administering the prescribed dose

A client undergoes a bowel resection. When assessing the client 4 hours postoperatively, the nurse identifies which finding as an early sign of shock?
1
Respirations of 10
2
Urine output of 30 ml/hour
3
Lethargy
4
Restlessness

Restlessness

What are the clinical indicators that a nurse expects when an intravenous (IV) line has infiltrated? (Select all that apply.)
1
Heat
2
Pallor
3
Edema
4
Decreased flow rate
5
Increased blood pressure

Pallor Edema Decreased flow rate

The health care provider prescribes a low-fat, 2-gram sodium diet for a client with hypertension. The nurse should explain that the purpose of restricting sodium is to:
1
Chemically stimulate the loop of Henle
2
Diminish the thirst response of the client
3
Prevent reabsorption of water in the distal tubules
4
Cause fluid to move toward the interstitial compartment

Prevent reabsorption of water in the distal tubules

An 89-year-old client with osteoporosis is admitted to the hospital with a compression fracture of the spine. The nurse identifies that a factor of special concern when caring for this client is the client’s:
1
Irritability in response to deprivation
2
Decreased ability to recall recent facts
3
Inability to maintain an optimal level of functioning
4
Gradual memory loss resulting from change in environment

Inability to maintain an optimal level of functioning

Health promotion efforts with the chronically ill client should include interventions related to primary prevention. What should this include?
1
Encouraging daily physical exercise
2
Performing yearly physical examinations
3
Providing hypertension screening programs
4
Teaching a person with diabetes how to prevent complications

Encouraging daily physical excercise

Several recently licensed practical nurses are discussing whether they should purchase personal professional liability insurance. Which statement indicates the most accurate information about professional liability insurance?
1
"If you have liability insurance, you are more likely to be sued."
2
"Your employer provides you with the liability insurance you will need."
3
"Liability insurance is not available for nursing professionals working in a hospital."
4
"Personal liability insurance offers representation if the State Board of Nursing files charges against you."

"Personal liability insurance offers representation if the State Board of Nursing files charges against you."

A nurse assesses a client’s serum electrolyte levels in the laboratory report. What electrolyte in intracellular fluid should the nurse consider most important?
1
Sodium
2
Calcium
3
Chloride
4
Potassium

Potassium

While caring for a client with a Hemovac portable wound drainage system, the nurse observes that the collection container is half full. The nurse empties the container. What is the next nursing intervention?
1
Encircle the drainage on the dressing.
2
Irrigate the suction tube with sterile saline.
3
Clean the drainage port with an alcohol wipe.
4
Compress the container before closing the port.

Compress the container before closing the port

An 80-year-old female is admitted to the hospital because of complications associated with severe dehydration. The client’s daughter asks the nurse how her mother could have become dehydrated because she is alert and able to care for herself. The nurse’s best response is:
1
"The body’s fluid needs decrease with age because of tissue changes."
2
"Access to fluid may be insufficient to meet the daily needs of the older adult."
3
"Memory declines with age, and the older adult may forget to ingest adequate amounts of fluid."
4
"The thirst reflex diminishes with age, and therefore the recognition of the need for fluid is decreased."

The thirst reflex diminishes with age, and therefore the recognition of the need for fluid is decreased

What is a basic concept associated with rehabilitation that the nurse should consider when formulating discharge plans for clients?
1
Rehabilitation needs are met best by the client’s family and community resources.
2
Rehabilitation is a specialty area with unique methods for meeting clients’ needs.
3
Immediate or potential rehabilitation needs are exhibited by clients with health problems.
4
Clients who are returning to their usual activities following hospitalization do not require rehabilitation.

Immediate or potential rehabilitation needs are exhibited by clients with health problems.

A nurse educator is presenting information about the nursing process to a class of nursing students. What definition of the nursing process should be included in the presentation?
1
Procedures used to implement client care.
2
Sequence of steps used to meet the client’s needs.
3
Activities employed to identify a client’s problem.
4
Mechanisms applied to determine nursing goals for the client

Sequence of steps used to meet the client’s needs.

Which nursing action is confidential and protected from legal action?
1
Providing health teaching regarding family planning.
2
Offering first aid at the scene of an automobile collision.
3
Reporting incidents of suspected child abuse to the appropriate authorities.
4
Administering resuscitative measures to an unconscious child pulled from a swimming pool.

Reporting incidents of suspected child abuse to the appropriate authorities.

A nurse is evaluating the appropriateness of a family member’s initial response to grief. What is the most important factor for the nurse to consider?
1
Personality traits
2
Educational level
3
Cultural background
4
Past experiences with death

Cultural background

What clinical finding indicates to the nurse that a client may have hypokalemia?
1
Edema
2
Muscle spasms
3
Kussmaul breathing
4
Abdominal distention

Abdominal distention

A client is being discharged from the hospital with an indwelling urinary catheter. The client asks about the best way to prevent infection and keep the catheter clean. Which would be appropriate for the nurse to include in the client teaching?
1
Once a day, clean the tubing with a mild soap and water, starting at the drainage bag and moving toward the insertion site.
2
After cleaning the catheter site, it is important to keep the foreskin pushed back for 30 minutes to ensure adequate drying.
3
Clean the insertion site daily using a solution of one part vinegar to two parts water.
4
Change the drainage bag at least once a week as needed.

Change the drainage bag at least once a week as needed

A nurse assesses drainage on a surgical dressing and documents the findings. Which documentation is most informative?
1
"Moderate amount of drainage."
2
"No change in drainage since yesterday."
3"
A 10-mm-diameter area of drainage at 1900 hours."
4
"Drainage is doubled in size since last dressing change."

A 10-mm-diameter area of drainage at 1900 hours."

A nurse is preparing to administer an ophthalmic medication to a client. What techniques should the nurse use for this procedure? (Select all that apply.)
1
Clean the eyelid and eyelashes.
2
Place the dropper against the eyelid.
3
Apply clean gloves before beginning of procedure.
4
Instill the solution directly onto cornea.
5
Press on the nasolacrimal duct after instilling the solution.

Clean the eyelid and eyelashes. Apply clean gloves before beginning of procedure. Press on the nasolacrimal duct after instilling the solution.

A client is being treated for Influenza A (H1N1). The nurse has provided instructions to the client about how to decrease the risk of transmission to others. Which patient statement indicates a need for further instruction/clarification?
1
"I should practice respiratory hygiene/cough etiquette."
2
"I should avoid contact with the elderly or children."
3
"I should obtain a pneumococcal vaccination each year."
4
"I should allow visitors for short periods of time only."

I should obtain a pneumococcal vaccination each year.

After gastric surgery a client has a nasogastric tube in place. What should the nurse do when caring for this client?
1
Monitor for signs of electrolyte imbalance.
2
Change the tube at least once every 48 hours.
3
Connect the nasogastric tube to high continuous suction.
4
Assess placement by injecting 10 mL of water into the tube.

Monitor for signs of electrolyte imbalance.

A nurse is supportive of a child receiving long-term rehabilitation in the home rather than in a health care facility. Why is living with the family so important to a child’s emotional development?
1
It provides rewards and punishment.
2
The child’s development is supported.
3
It reflects the mores of a larger society.
4
It is where child’s identity and roles are learned

It is where child’s identity and roles are learned

The nurse is having difficulty understanding a client’s decision to have hospice care rather than an extensive surgical procedure. Which ethical principle does the client’s behavior illustrate?
1
Justice
2
Veracity
3
Autonomy
4
Beneficence

Autonomy

A client receiving intravenous vancomycin (Vancocin) reports ringing in both ears. Which initial action should the nurse take?
1
Notify the primary healthcare provider.
2
Consult an audiologist.
3
Stop the infusion.
4
Document the finding and continue to monitor the client.

Stop the infusion

A client with a diagnosis of uncontrolled diabetes began receiving Lasix (Furosemide) two days ago. The nurse reviews the morning lab results and discovers that the client’s potassium level is 2.8 mEq/L. What is the most appropriate action for the nurse to take?
1
Hold the morning dose of the diuretic and have the lab repeat the test.
2
Continue to monitor the level to ensure that it stays within the normal limits.
3
Notify the primary healthcare provider of the result, which is critically low.
4
Anticipate a prescription for an increase in the dosage of the Lasix.

Notify the primary healthcare provider of the result, which is critically low

The nurse is discussing discharge plans with a client who had a myocardial infarction. The client states, "I’m worried about going home." The nurse responds, "Tell me more about this." What interviewing technique did the nurse use?
1
Exploring
2
Reflecting
3
Refocusing
4
Acknowledging

Exploring

A nurse takes into consideration that the key factor in accurately assessing how a client will cope with body image changes is the:
1
Suddenness of the change
2
Obviousness of the change
3
Extent of the body changes
4
Perception of the body changes

Perception of the body changes

The nurse receives a report on a newly admitted client who is positive for Clostridium difficile. Which category of isolation would the nurse implement for this client?
1
Airborne precautions
2
Droplet precautions
3
Contact precautions
4
Protective environment

Contact precautions

The nurse manager is planning to assign unlicensed assistive personnel (UAP) to care for clients. What care can be delegated on a medical-surgical unit to UAP? (Select all that apply.)
1
Performing a bed bath for a client on bed rest.
2
Evaluating the effectiveness of acetaminophen and codeine (Tylenol #3).
3
Obtaining an apical pulse rate before oral digoxin (Lanoxin) is administered.
4
Assisting a client who has patient-controlled analgesia (PCA) to the bathroom.
5
Assessing the wound integrity of a client recovering from an abdominal laparotomy.

Performing a bed bath for a client on bed rest Assisting a client who has patient-controlled analgesia (PCA) to the bathroom.

When caring for a client with pneumonia, which nursing intervention is the highest priority?
1
Increase fluid intake.
2
Employ breathing exercises and controlled coughing.
3
Ambulate as much as possible.
4
Maintain an NPO status.

Employ breathing exercises and controlled coughing

After changing a dressing that was used to cover a draining wound on a client with Vancomycin Resistant Enterococcus (VRE), the nurse should take which step to ensure proper disposal of soiled dressing?
1
Place the dressing in the bedside trash can.
2
Place the dressing in a red bag/hazardous materials bag.
3
Contact Environmental Services personnel to pick up the dressing.
4
Transport the dressing to the laboratory to be placed in the incinerator.

Place the dressing in a red bag/hazardous materials bag.

The unlicensed assistive person (UAP) assigned to the 7 am shift has not been coming to work until 8 am. Nursing care is delayed and assignments are started late. What is the most appropriate action by the charge nurse/team leader?
1
Discuss the issue with a friend from another unit
2
Remind the UAP of the expected start time
3
Report the problem to the Human Resources department
4
Document the information before discussing it with the UAP

Document the information before discussing it with the UAP

A nurse applies a heating pad to a client’s buttocks. Upon removal of the heating pad, the nurse discovers that the client has received burns due to incorrect settings when use of the heating pad was initiated. Which principle would legally apply?
1
No one could be held liable for new equipment.
2
The nurse could be held liable for the injury that occurred.
3
The nurse did what a reasonable, prudent nurse would do.
4
The manufacturer is liable for new equipment.

The nurse could be held liable for the injury that occurred.

The nurse caring for a client with a systemic infection is aware that the assessment finding that is most indicative of a systemic infection is:
1
White blood cell (WBC) count of 8200/mm3
2
Bilateral 3+ pitting pedal edema
3
Oral temperature of 101.3º F
4
Pale skin and nail beds

Oral temperature of 101.3º F

A client with a fractured tibia and fibula is to be discharged from the emergency department with a right leg cast and crutches. In addition to the technical aspects of crutch walking, the nurse should teach the client to:
1
Decrease calcium intake
2
Remove loose rugs from the environment
3
Avoid taking showers until the cast is removed
4
Increase weight bearing on the injured leg gradually

Remove loose rugs from the environment

The nurse is caring for a client admitted with chronic obstructive pulmonary disease (COPD). The nurse should monitor the results of which laboratory test to evaluate the client for hypoxia?
1
Red blood cell count
2
Sputum culture
3
Arterial blood gas
4
Total hemoglobin

Arterial blood gas

The nurse is having difficulty understanding a client’s decision to have hospice care rather than an extensive surgical procedure. Which ethical principle does the client’s behavior illustrate?
1
Justice
2
Veracity
3
Autonomy
4
Beneficence

Autonomy

A nurse who promotes freedom of choice for clients in decision-making best supports which principle?
1
Justice
2
Autonomy
3
Beneficence
4
Paternalism

Autonomy

A client spends several minutes making negative comments to the nurse about numerous aspects of the hospital stay. What is the nurse’s best initial response?
1
Describe the purpose of different hospital therapies to decrease the client’s anxiety.
2
Explain that becoming so upset does not allow the client to get much-needed rest.
3
Refocus the conversation on the client’s fears, frustrations, and anger about the condition.
4
Permit the client to release feelings and then leave the room to allow the client to regain composure.

Refocus the conversation on the client’s fears, frustrations, and anger about the condition.

What should the nurse consider when obtaining an informed consent from a 17-year-old adolescent?
1
If the client is allowed to give consent.
2
The client cannot make informed decisions about health care.
3
If the client is permitted to give voluntary consent when parents are not available.
4
The client probably will be unable to choose between alternatives when asked to consent.

If the client is allowed to give consent.

When changing the soiled bed linens of a client with a wound that is draining seropurulent material, what personal protective equipment (PPE) is most essential for the nurse to wear?
1
Mask
2
Clean gloves
3
Sterile gloves
4
Shoe covers

Sterile gloves

A client with a history of chronic obstructive pulmonary disease (COPD) is admitted with acute bronchopneumonia. The client is in moderate respiratory distress. The nurse should place the client in what position to enhance comfort?
1
Side lying position with head elevated 45 degrees
2
Sim’s position with head elevated 90 degrees
3
Semi-Fowler’s position with legs elevated
4
High-Fowler’s position using the bedside table as an arm rest

High-Fowler’s position using the bedside table as an arm rest

A nurse is caring for a client diagnosed with Methicillin-Resistant Staphylococcus Aureus (MRSA) in the urine. The health care provider orders an indwelling urinary catheter to be inserted. Which precaution should the nurse take during this procedure?
1
Droplet precautions
2
Reverse isolation
3
Surgical asepsis
4
Medical asepsis

Surgical asepsis

When changing the soiled bed linens of a client with a wound that is draining seropurulent material, what personal protective equipment (PPE) is most essential for the nurse to wear?
1
Mask
2
Clean gloves
3
Sterile gloves
4
Shoe covers

Clean gloves

The nurse is preparing discharge instructions for a client who was prescribed enalapril maleate (Vasotec) for treatment of hypertension. Which is appropriate for the nurse to include in the client’s teaching?
1
Do not change positions suddenly.
2
Light-headedness is a common adverse effect that need not be reported.
3
The medication may cause a sore throat for the first few days.
4
Schedule blood tests weekly for the first 2 months

Do not change positions suddenly

A 50-year-old client being seen for a routine physical asks why a stool specimen for occult blood testing has been prescribed when there is no history of health problems. What is an appropriate nursing response?
1
"You will need to ask your healthcare provider; it is not part of the usual tests for people your age."
2
"There must be concern of a family history of colon cancer; that is a primary reason for an occult blood stool test."
3
"It is performed routinely starting at your age as part of an assessment for colon cancer."
4
"There must have been a positive finding after a digital rectal examination performed by your healthcare provider."

It is performed routinely starting at your age as part of an assessment for colon cancer

An adolescent that had an inguinal hernia repair is being prepared for discharge home. The nurse provides instructions about resumption of physical activities. Which statement by the adolescent indicates that the client understands the instructions?
1
"I can ride my bike in about a week."
2
"I don’t have to go to gym class for 3 months."
3
"I can’t perform any weightlifting for at least 3 weeks."
4
"I can never participate in football again."

I can’t perform any weightlifting for at least 3 weeks

A client has received instructions to take 650 mg aspirin (ASA) every 6 hours as needed for arthritic pain. What should the nurse include in the client’s medication teaching? (Select all that apply.)
1
Take the aspirin with meals or a snack.
2
Make an appointment with a dentist if bleeding gums develop.
3
Do not chew enteric-coated tablets.
4
Switch to Tylenol (acetaminophen) if tinnitus occurs.
5
Report persistent abdominal pain.

Take the aspirin with meals or a snack Do not chew enteric-coated tablets Report persistent abdominal pain

A client is admitted to the hospital for an elective surgical procedure. The client tells a nurse about the emotional stress of recently disclosing being a homosexual to family and friends. What is the nurse’s first consideration when planning care?
1
Exploring the client’s emotional conflict
2
Identifying personal feelings toward this client
3
Planning to discuss this with the client’s family
4
Developing a rapport with the client’s health care provider

Identifying personal feelings toward this client.

Following a surgery on the neck, the client asks the nurse why the head of the bed is up so high. The nurse should tell the client that the high-Fowler position is preferred for what reason?
1
To avoid strain on the incision
2
To promote drainage of the wound
3
To provide stimulation for the client
4
To reduce edema at the operative site

To reduce edema at the operative site

A client that is scheduled for a surgical resection of the colon and creation of a colostomy for a bowel malignancy asks why preoperative antibiotics have been prescribed. The nurse explains that the primary purpose is to:
1
Decrease peristalsis.
2
Minimize electrolyte imbalance.
3
Decrease bacteria in the intestines.
4
Treat inflammation caused by the malignancy.

Decrease bacteria in the intestinves

The nurse is preparing discharge instructions for a client who has begun to demonstrate signs of early Alzheimer’s dementia. The client lives alone. The client’s adult children live nearby. According to the prescribed medication regimen the client is to take medications six times throughout the day. What is priority nursing intervention to assist the client with compliance with medication-taking?
1
Contact the client’s children and ask them to hire a private duty aide who will provide round-the-clock care.
2
Develop a chart for the client, listing the times the medication should be taken.
3
Contact the primary healthcare provider and discuss the possibility of simplifying the medication regimen.
4
Instruct the client and client’s children to put medications in a weekly pill organizer.

Contact the primary healthcare provider and discuss the possibility of simplifying the medication regimen

Which nursing intervention is most appropriate for a client in skeletal traction?
1
Add and remove weights as the client desires.
2
Assess the pin sites at least every shift and as needed.
3
Ensure that the knots in the rope are tied to the pulley.
4
Perform range of motion to joints proximal and distal to the fracture at least once a day.

Assess the pin sites at least every shift and as needed

What is a nurse’s responsibility when administering prescribed opioid analgesics? (Select all that apply.)
1
Count the client’s respirations.
2
Document the intensity of the client’s pain.
3
Withhold the medication if the client reports pruritus.
4
Verify the number of doses in the locked cabinet before administering the prescribed dose.
5
Discard the medication in the client’s toilet before leaving the room if the medication is refused.

Count the client’s respirations Document the intensity of the client’s pain Verify the number of doses in the locked cabinet before administering the prescribed dose

When providing preoperative teaching, the nurse should focus primarily on:
1
Helping the client and family decide if surgery is necessary.
2
Providing emotional support to the client and family.
3
Giving minute-by-minute details of the surgery to the client and family.
4
Providing general information to reduce client and family anxiety.

Providing general information to reduce client and family anxiety

Based on the client’s reported pain level, the nurse administers 8 mg of the prescribed morphine. The medication is available in a 10 mg syringe. Wasting of the remaining 2 mg of morphine should be done by the nurse and a witness. It is most appropriate for the nurse to ask which member of the health care team to be the witness?
1
Nursing supervisor
2
Licensed practical nurse (LPN)
3
Client’s health care provider
4
Designated nursing assistant

Licensed practical nurse (LPN)

The nurse is caring for a client that is hyperventilating. The nurse recalls that the client is at risk for:
1
Respiratory acidosis
2
Respiratory alkalosis
3
Respiratory compensation
4
Respiratory decompensation

Respiratory alkalosis

When caring for a client with varicella and disseminated herpes zoster, the nurse should implement which types of precautions? (Select all that apply.)
1
Airborne
2
Contact
3
Droplet
4
Hazardous Wastes
5
Standard

Airborne Contact Standard ACS

A client admitted to the hospital with a diagnosis of malabsorption syndrome exhibits signs of tetany. The nurse concludes that the tetany was precipitated by the inadequate absorption of which electrolyte?
1
Sodium
2
Calcium
3
Potassium
4
Phosphorus

Calcium

A client with arthritis increases the dose of ibuprofen (Motrin, Advil) to abate joint discomfort. After several weeks the client becomes increasingly weak. The client is admitted to the hospital and is diagnosed with severe anemia. What clinical indicators does the nurse expect to identify when performing an admission assessment? (Select all that apply.)
1
Melena
2
Tachycardia
3
Constipation
4
Clay-colored stools
5
Painful bowel movements

Melena Tachycardia

A pain scale of 1 to 10 is used by a nurse to assess a client’s degree of pain. The client rates the pain as an 8 before receiving an analgesic and a 7 after being medicated. What conclusion should the nurse make regarding the client’s response to pain medication?
1
Client has a low pain tolerance.
2
Medication is not adequately effective.
3
Medication has sufficiently decreased the pain level.
4
Client needs more education about the use of the pain scale

Medication is not adequately effective

The nurse has provided instructions about back safety to a client. Which client statement indicates understanding of the instructions?
1
"I should carry objects about 18 inches from my body."
2
"I should sleep on my stomach with a firm mattress."
3
"I should carry objects close to my body."
4
"I should pull rather than push when moving heavy objects."

I should carry objects close to my body

What are the clinical indicators that a nurse expects when an intravenous (IV) line has infiltrated? (Select all that apply.)
1
Heat
2
Pallor
3
Edema
4
Decreased flow rate
5
Increased blood pressur

Pallor Edema Decreased flow rate

A nurse provides discharge teaching related to intermittent urinary self-catheterization to a client with a new spinal cord injury. Which instruction is most important for the nurse to include?
1
"Wear sterile gloves when doing the procedure."
2
"Wash your hands before performing the procedure."
3
"Perform the self-catheterization every 12 hours."
4
"Dispose of the catheter after you have catheterized yourself."

Wash your hands before performing the procedure

To prevent thrombophlebitis in the immediate postoperative period, which action is most important for a nurse to include in the client’s plan of care?
1
Increase fluid intake.
2
Restrict fluids.
3
Encourage early mobility.
4
Elevate the knee gatch of the bed.

Encourage early mobility

A nurse is caring for a client with hemiplegia who is frustrated. How can the nurse motivate the client toward independence?
1
Establish long-range goals for the client.
2
Identify errors that the client can correct.
3
Reinforce success in tasks accomplished.
4
Demonstrate ways to promote self-reliance.

Reinforce success in tasks accomplished

The nurse creates a plan of care for a client with a risk of infection. Which is the most desirable expected outcome for the client?
1
All nursing functions will be completed by discharge.
2
All invasive intravenous lines will remain patent.
3
The client will remain awake, alert, and oriented at all times.
4
The client will be free of signs and symptoms of infection by discharge

The client will be free of signs and symptoms of infection by discharge

When assessing an obese client, a nurse observes dehiscence of the abdominal surgical wound with evisceration. The nurse places the client in the low-Fowler position with the knees slightly bent and encourages the client to lie still. What is the next nursing action?
1
Obtain the vital signs.
2
Notify the health care provider.
3
Reinsert the protruding organs using aseptic technique.
4
Cover the wound with a sterile towel moistened with normal saline

Cover the wound with a sterile towel moistened with normal saline

A health care provider prescribes famotidine (Pepcid) and magnesium hydroxide/aluminum hydroxide (Maalox) for a client with a peptic ulcer. The nurse should teach the client to take the Maalox at what time?
1
Only at bedtime, when famotidine is not taken.
2
Only if famotidine is ineffective.
3
At the same time as famotidine, with a full glass or water.
4
One hour before or two hours after famotidine.

One hour before or two hours after famotidine

What is a nurse’s responsibility when administering prescribed opioid analgesics? (Select all that apply.)
1
Count the client’s respirations.
2
Document the intensity of the client’s pain.
3
Withhold the medication if the client reports pruritus.
4
Verify the number of doses in the locked cabinet before administering the prescribed dose.
5
Discard the medication in the client’s toilet before leaving the room if the medication is refused.

Count the client’s respirations Document the intensity of the client’s pain Verify the number of doses in the locked cabinet before administering the prescribed dose

A client’s serum potassium level has increased to 5.8 mEq/L. What action should the nurse implement first?
1
Call the laboratory to repeat the test.
2
Take vital signs and notify the charge nurse or health care provider.
3
Inform the cardiac arrest team to place them on alert.
4
Take an electrocardiogram and have lidocaine available

Take vital signs and notify the charge nurse or health care provider

As a nurse prepares an older adult client for sleep, actions are taken to help reduce the likelihood of a fall during the night. What nursing action is most appropriate when targeting older adults’ most frequent cause of falls?
1
Moving the client’s bedside table closer to the bed.
2
Encouraging the client to take an available sedative.
3
Instructing the client to call the nurse before going to the bathroom.
4
Assisting the client to telephone home to say goodnight to the spouse

Instructing the client to call the nurse before going to the bathroom

A nurse assesses a client with dry and brittle hair, flaky skin, a beefy-red tongue and bleeding gums. The nurse recognizes that these clinical manifestations are most likely a result of:
1
A food allergy.
2
Noncompliance with medications.
3
Side effects from medications.
4
A nutritional deficiency.

A nutritional deficiency

The nurse reviews a medical record and is concerned that the client may develop hyperkalemia. Which disease increases the risk of hyperkalemia?
1
Crohn’s
2
Cushing’s
3
End-stage renal
4
Gastroesophageal reflux

End-stage renal

A client’s intravenous (IV) infusion infiltrates. The nurse concludes that what is most likely the cause of the infiltration?
1
Excessive height of the IV bag
2
Failure to secure the catheter adequately
3
Contamination during the catheter insertion
4
Infusion of a chemically irritating medication

Failure to secure the catheter adequately

The nurse providing post-procedure care to a client who had a cardiac catheterization through the femoral artery discovers a large amount of blood under the client’s buttocks. After donning gloves, which action should the nurse take first?
1
Apply pressure to the site.
2
Obtain vital signs.
3
Change the client’s gown and bed linens.
4
Assess the catheterization site

Assess the catheterization site

The hospital’s policy requires two nurses to supervise the wasting of excess opioid solutions. The nurse draws up the prescribed dose and then requests that another nurse witness wasting of the remaining medication. The second nurse states that there is no time to observe the wasting of the medication, enters the identification to serve as the witness, and leaves the area. What is the appropriate action for the first nurse to take?
1
Waste the appropriate amount of medication and administer the appropriate dose to the client.
2
Accept the second nurse’s identification as the witness but ask another nurse to observe the actual wasting.
3
Cancel the process and ask another nurse to serve as the witness and to observe the wasting of the medication.
4
Insist that the second nurse re-enter the area to actually observe the wasting of the medication.

Cancel the process and ask another nurse to serve as the witness and to observe the wasting of the medication.

What does a nurse consider the most significant influence on many clients’ perception of pain when interpreting findings from a pain assessment?
1
Age and sex
2
Physical and physiological status
3
Intelligence and economic status
4
Previous experience and cultural values

Previous experience and cultural values

A health care provider prescribes simvastatin (Zocor) 20 mg daily for elevated cholesterol and triglyceride levels for a female client. Which is most important for the nurse to teach when the client initially takes the medication?
1
Take the medication with breakfast.
2
Have liver function tests every six months.
3
Wear sunscreen to prevent photosensitivity reactions.
4
Inform the health care provider if the client wishes to become pregnant.

Inform the health care provider if the client wishes to become pregnant

A client is admitted with metabolic acidosis. The nurse considers that two body systems interact with the bicarbonate buffer system to preserve healthy body fluid pH. What two body systems should the nurse assess for compensatory changes?
1
Skeletal and nervous
2
Circulatory and urinary
3
Respiratory and urinary
4
Muscular and endocrine

Respiratory and urinary

The nurse manager of the unit comes to work obviously intoxicated. The staff nurse’s ethical obligation is to:
1
Call the security guard
2
Tell the nurse manager to go home
3
Have the supervisor validate the observation
4
Offer the nurse manager a large cup of coffee

Have the supervisor validate the observation

A nurse is reviewing a plan of care for a client who was admitted with dehydration as a result of prolonged watery diarrhea. Which prescription should the nurse question?
1
Oral psyllium (Metamucil)
2
Oral potassium supplement
3
Parenteral half normal saline
4
Parenteral albumin (Albuminar)

Parenteral albumin (Albuminar)

A nurse is providing colostomy care to a client with a nosocomial infection caused by methicillin-resistant Staphylococcus aureus (MRSA). Which personal protective equipment (PPE) should the nurse use? (Select all that apply.)
1
Gloves
2
Gown
3
Mask
4
Goggles
5
Shoe covers
6
Hair bonnet

Gloves Gown Goggles

A physician orders a urinalysis for a client with an indwelling catheter. To ensure that an appropriate specimen is obtained, the nurse would obtain the specimen from which site?
1
tubing injection port
2
distal end of the tubing
3
urinary drainage bag
4
catheter insertion site

tubing injection port

A nurse reviews a medical record of a client with ascites. What does the nurse identify that may be causing the ascites?
1
Portal hypotension
2
Kidney malfunction
3
Decreased liver function
4
Decreased production of potassium

Decreased liver function

A nurse instructs a client to breathe deeply to open collapsed alveoli. What should the nurse include in the explanation of the relationship between alveoli and improved oxygenation?
1
"The alveoli need oxygen to live."
2
"The alveoli have no direct effect on oxygenation."
3
"Collapsed alveoli increase oxygen demands."
4
"Oxygen is exchanged for carbon dioxide in the alveolar membrane."

Oxygen is exchanged for carbon dioxide in the alveolar membrane

A health care provider prescribes a standard walker (pick-up walker with rubber tips on all four legs). The nurse identifies what clinical findings that indicate the client is capable of using a standard walker?
1
Weak upper arm strength and impaired stamina
2
Weight bearing as tolerated and unilateral paralysis
3
Partial weight bearing on the affected extremity and kyphosis
Correct4
Strong upper arm strength and non-weight bearing on the affected extremity

Strong upper arm strength and non-weight bearing on the affected extermity

A nurse is caring for a client with diarrhea. The nurse anticipates a decrease in which clinical indicator?
1
Pulse rate
2
Tissue turgor
3
Specific gravity
4
Body temperature

Tissue Turgor

A client’s chest tube has accidentally dislodged. What is the nursing action of highest priority?
1
Place the client in a left side-lying position.
2
Apply oxygen via non-rebreather mask.
3
Apply a petroleum gauze dressing over the site.
4
Prepare to reinsert a new chest tube

Apply a petroleum gauze dressing over the site

A client has received instructions to take 650 mg aspirin (ASA) every 6 hours as needed for arthritic pain. What should the nurse include in the client’s medication teaching? (Select all that apply.)
1
Take the aspirin with meals or a snack.
2
Make an appointment with a dentist if bleeding gums develop.
3
Do not chew enteric-coated tablets.
4
Switch to Tylenol (acetaminophen) if tinnitus occurs.
5
Report persistent abdominal pain.

Take the aspirin with meals or snack Do not chew enteric-coated tablets Report persistent abdominal pain

A client is receiving albuterol (Proventil) to relieve severe asthma. For which clinical indicators should the nurse monitor the client? (Select all that apply.)
1
Tremors
2
Lethargy
3
Palpitations
4
Visual disturbances
5
Decreased pulse rate

Tremors Palpitations

female client explains to the nurse that she sleeps until noon every day and takes frequent naps during the rest of the day. What should the nurse do initially?
Incorrect1
Encourage her to exercise during the day
2
Arrange a referral for a thorough medical evaluation
3
Explain that this behavior is an attempt to avoid facing daily responsibilities
4
Identify that the client is describing clinical findings associated with narcolepsy

Arrange a referral for a thorough medical evaluation

A client is scheduled to receive phenytoin (Dilantin) 100 mg orally at 6 PM but is having difficulty swallowing capsules. What method should the nurse use to help the client take the medication?
1
Sprinkle the powder from the capsule into a cup of water.
2
Insert a rectal suppository containing 100 mg of phenytoin.
3
Contact the prescriber to determine if a change to a suspension form would be possible.
4
Obtain a change in the administration route to allow an intramuscular (IM) injection

Contact the prescriber to determine if a change to a suspension form would be possible

A client with Type I Diabetes complains of hunger, thirst, tiredness, and frequent urination. Based on these findings, the nurse should take what action?
1
Notify the physician immediately about the client’s symptoms.
2
Determine the client’s blood glucose level.
3
Administer the client’s prescribed insulin.
4
Give the client a peanut butter and graham cracker snack

Determine the client’s blood glucose level

A nurse is transcribing a practitioner’s orders for a group of clients. Which order should the nurse clarify with the practitioner?
1
Discharge in am
2
Blood glucose monitoring ac and bedtime
3
Erythropoietin (Procrit) 6000 units subcutaneously TIW
4
Dalteparin (Fragmin) 5000 international units Sub-Q BID

Erythropoietin (Procrit) 6000 units subcutaneously TIW

The nurse is providing post-procedure care for a client that had a liver biopsy. To prevent hemorrhage, it is the nurse’s highest priority to place the client in what position?
1
Prone
2
High-Fowler’s
3
On the right side
4
Trendelenburg

On the right side

A client has been admitted with a diagnosis of intractable vomiting and can only tolerate sips of water. The initial blood work shows a sodium level of 122 mEq/L and a potassium level of 3.6 mEq/L. Based on the lab results and symptoms, what is the client experiencing?
1
Hypernatremia
2
Hyponatremia
3
Hyperkalemia
4
Hypokalemia

Hyponatremia

A client is scheduled to receive conscious sedation during a colonoscopy. The client asks the nurse, "How will they ‘knock me out’ for this procedure?" Which answer by the nurse correctly describes the route of administration for conscious sedation?
1
"You will receive the anesthesia through a face mask."
2
"You will receive medication through an intravenous catheter."
3
"We will give you an oral medication about one hour before the procedure."
4
"The nurse anesthetist will inject the medication into the epidural space of your spine.

You will receive medication through an intravenous catheter

The nurse teaching a health awareness class identifies which situation as being the highest risk factor for the development of a deep vein thrombosis (DVT)?
1
Pregnancy
2
Inactivity
3
Aerobic exercise
4
Tight clothing

Inactivity

A client with an abdominal wound infected with methicillin-resistant Staphylococcus aureus (MRSA) is scheduled for a computed tomography (CT) scan of the abdomen. To ensure client and visitor safety during transport, the nurse should implement which precaution?
1
No special precautions are required.
2
Cover the infected site with a dressing.
3
Drape the client with a covering labeled as biohazardous.
4
Place a surgical mask on the client

Cover the infected site with a dressing

A client is being treated for Influenza A (H1N1). The nurse has provided instructions to the client about how to decrease the risk of transmission to others. Which patient statement indicates a need for further instruction/clarification?
1
"I should practice respiratory hygiene/cough etiquette."
2
"I should avoid contact with the elderly or children."
3
"I should obtain a pneumococcal vaccination each year."
4
"I should allow visitors for short periods of time only."

I should obtain pneumococcal vaccination each year

A client that is scheduled for a surgical resection of the colon and creation of a colostomy for a bowel malignancy asks why preoperative antibiotics have been prescribed. The nurse explains that the primary purpose is to:
1
Decrease peristalsis.
2
Minimize electrolyte imbalance.
3
Decrease bacteria in the intestines.
4
Treat inflammation caused by the malignancy

Decrease bacteria in the intestines

A client with rheumatoid arthritis does not want the prescribed cortisone and informs the nurse. Later, the nurse attempts to administer cortisone. When the client asks what the medication is, the nurse gives an evasive answer. The client takes the medication and later discovers that it was cortisone. The client states an intent to sue. What factors in this situation must be considered in a legal action? (Select all that apply.)
1
Clients have a right to refuse treatment.
2
Nurses are required to answer clients truthfully.
3
The health care provider should have been notified.
4
The client had insufficient knowledge to make such a decision.
5
Legally prescribed medications are administered despite a client’s objections.

Clients have a right to refuse treatment Nurses are required to answer clients truthfully The health care provider should have been notified

A client is admitted to the hospital and benazepril hydrochloride (Lotensin) is prescribed for hypertension. Which is an appropriate nursing action for clients taking this medication?
1
Monitor the EEG.
2
Assess for dizziness.
3
Administer the drug after meals.
4
Assess for dark, tarry stools

Assess for dizziness

A health care provider prescribes digoxin (Lanoxin) for a client. The nurse teaches the client to be alert for which common early indication of digoxin toxicity?
1
Nausea
2
Urticaria
3
Photophobia
4
Yellow vision

Nausea

At the conclusion of visiting hours, the parent of a 14-year-old adolescent scheduled for orthopedic surgery the next day hands the nurse a bottle of capsules and says, "These are for my child’s allergy. Will you be sure my child takes one about 9 PM tonight?" What is the nurse’s best response?
1
"I will give one capsule tonight before bedtime."
2
"I will get a prescription so that the medicine can be taken."
3
"Does your health care provider know about your child’s allergy?"
4
"Did you ask your health care provider if your child should have this tonight?

"I will get a prescription so that the medicine can be taken."

A nurse is providing morning hygiene to a bedridden client who was admitted for exacerbation of chronic obstructive pulmonary disease (COPD). What is the priority nursing intervention when the client becomes short of breath during the care?
1
Obtain a pulse oximeter to determine the client’s oxygen saturation level.
2
Put the client in a high-Fowler’s position.
3
Darken the lights and provide a rest period of at least 15 minutes.
4
Continue the hygiene activities while reassuring the client

Put the client in a high-Fowler’s position.

A nurse applies a cold pack to treat an acute musculoskeletal injury. Cold therapy decreases pain by:
1
Promoting analgesia and circulation
2
Numbing the nerves and dilating the blood vessels
3
Promoting circulation and reducing muscle spasms
4
Causing local vasoconstriction, preventing edema and muscle spasm

Causing local vasoconstriction, preventing edema and muscle spasm

When reviewing a drug to be administered, the nurse identifies that the package insert indicates that the Z-track injection technique should be used. Under what circumstance does the nurse expect that this technique will be necessary?
1
Volume of medication to be administered is large.
2
Medication is irritating to subcutaneous tissue and skin.
3
Injection site must be massaged after it is administered.
4
Procedure requires an air bubble to be drawn into the syringe.

Medication is irritating to subcutaneous tissue and skin

When monitoring fluids and electrolytes, the nurse recalls that the major cation-regulating intracellular osmolarity is:
1
Sodium
2
Potassium
3
Calcium
4
Calcitonin

Potassium

What clinical finding does a nurse anticipate when admitting a client with an extracellular fluid volume excess?
1
Rapid, thready pulse
2
Distended jugular veins
3
Elevated hematocrit level
4
Increased serum sodium level

Distended jugular veins

A nurse administers an intravenous solution of 0.45% sodium chloride. In what category of fluids does this solution belong?
1
Isotonic
2
Isomeric
3
Hypotonic
4
Hypertonic

Hypotonic

A nurse who is working on a medical-surgical unit receives a phone call requesting information about a client who has undergone surgery. The nurse observes that the client requested a do not publish ("DNP") order on any information regarding condition or presence in the hospital. What is the best response by the nurse?
1
"We have no record of that client on our unit. Thank you for calling."
2
"The new privacy laws prevent me from providing any client information over the phone."
3
"The client has requested that no information be given out. You’ll need to call the client directly."
4
"It is against the hospital’s policy to provide you with any information regarding any of our clients."

We have no record of that client on our unit. Thank you for calling." 2

A nurse is assigned to take care of a group of clients. Which client should the nurse see first?
1
A 2-yr-old male with diarrhea
2
A 35-yr-old male who is nauseated
3
A 40-yr-old female who has vomiting due to food poisoning
4
An 83-yr-old female whose last bowel movement was three days ago.

A 2-yr-old male with diarrhea

A nurse is taking care of a client who has severe back pain as a result of a work injury. What nursing considerations should be made when determining the client’s plan of care? (Select all that apply.)
1
Ask the client what is the client’s acceptable level of pain.
2
Eliminate all activities that precipitate the pain.
3
Administer the pain medications regularly around the clock.
4
Use a different pain scale each time to promote patient education.
5
Assess the client’s pain every 15 minutes

Ask the client what is the client’s acceptable level of pain. Administer the pain medications regularly around the clock.

What are the desired outcomes that the nurse expects when administering a nonsteroidal antiinflammatory drug (NSAID)? (Select all that apply.)
1
Diuresis
2
Pain relief
3
Antipyresis
4
Bronchodilation
5
Anticoagulation
6
Reduced inflammation

Pain reief Antipyresis Reduced inflammation

A nurse provides teaching for a client who is scheduled for a cholecystectomy. In the initial postoperative period, the nurse explains that the most important part of the treatment plan is:
1
Early ambulation
2
Coughing and deep breathing
3
Wearing anti-embolic elastic stockings
4
Maintenance of a nasogastric tube

Coughing and deep breathing

A primary nurse receives prescriptions for a newly admitted client and has difficulty reading the health care provider’s writing. Who should the nurse ask for clarification of this prescription?
1
Nurse practitioner
2
House health care provider that is on-call
3
Health care provider who wrote the prescription
4
Nurse manager familiar with the health care provider’s writing

Health care provider who wrote the prescription

A client has a paracentesis, and the health care provider removes 1500 mL of fluid. To monitor for a serious postprocedure complication, the nurse should assess for:
1
Dry mouth
2
Tachycardia
3
Hypertensive crisis
4
Increased abdominal distention

Tachycardia

A nurse is providing morning hygiene to a bedridden client who was admitted for exacerbation of chronic obstructive pulmonary disease (COPD). What is the priority nursing intervention when the client becomes short of breath during the care?
1
Obtain a pulse oximeter to determine the client’s oxygen saturation level.
2
Put the client in a high-Fowler’s position.
3
Darken the lights and provide a rest period of at least 15 minutes.
4
Continue the hygiene activities while reassuring the client

Put the client in a high-Fowler’s position

A client reports severe pain two days after surgery. After assessing the characteristics of the pain, which initial action should the nurse take next?
1
Encourage rest.
2
Obtain the vital signs.
3
Administer the prescribed analgesic.
4
Document the client’s pain response.

Obtain the vital signs

What clinical indicators should the nurse expect a client with hyperkalemia to exhibit? (Select all that apply.)
1
Tetany
2
Seizures
3
Diarrhea
4
Weakness
5
Dysrhythmias

Diarrhea Weakness Dysrhythmias

A client is receiving albuterol (Proventil) to relieve severe asthma. For which clinical indicators should the nurse monitor the client? (Select all that apply.)
1
Tremors
2
Lethargy
3
Palpitations
4
Visual disturbances
5
Decreased pulse rate

Tremors Palpitations

A nurse is caring for an elderly client with dementia who has developed dehydration as a result of vomiting and diarrhea. Which assessment best reflects the fluid balance of this client?
1
Skin turgor
2
Intake and output results
3
Client’s report about fluid intake
4
Blood lab results

Blood lab results

A client with Addison’s disease is receiving cortisone therapy. The nurse expects what clinical indicators if the client abruptly stops the medication? (Select all that apply.)
1
Diplopia
2
Dysphagia
3
Tachypnea
4
Bradycardia
5
Hypotension

Tachypnea Hypotension

What response should a nurse be particularly alert for when assessing a client for side effects of long-term cortisone therapy?
1
Hypoglycemia
2
Severe anorexia
3
Anaphylactic shock
4
Behavioral changes

Behavioral changes

A client with arthritis increases the dose of ibuprofen (Motrin, Advil) to abate joint discomfort. After several weeks the client becomes increasingly weak. The client is admitted to the hospital and is diagnosed with severe anemia. What clinical indicators does the nurse expect to identify when performing an admission assessment? (Select all that apply.)
1
Melena
2
Tachycardia
3
Constipation
4
Clay-colored stools
5
Painful bowel movements

Melena Tachycardia

Neomycin, 1 gram, is prescribed preoperatively for a client with cancer of the colon. The client asks why this is necessary. How should the nurse respond?
1
"It is used to prevent you from getting a bladder infection before surgery."
2
"It will decrease your kidney function and lessen urine production during surgery."
3
"It will kill the bacteria in your bowel and decrease the risk for infection after surgery."
4
"It is used to alter the body flora, which reduces spread of the tumor to adjacent organs."

"It will kill the bacteria in your bowel and decrease the risk for infection after surgery."

A nurse is assisting a client to transfer from the bed to a chair. What should the nurse do to widen the client’s base of support during the transfer?
1
Spread the client’s feet away from each other.
2
Move the client on the count of three.
3
Instruct the client to flex the muscles of the internal girdle.
4
Stand close to the client when assisting with the move

Spread the client’s feet away from each other.

A client has Clostridium difficile. The nurse is providing discharge instructions related to decreasing the risk of transmission to family members. What would be appropriate to include in the client’s teaching?
1
Increase fluids.
2
Increase fiber in the diet.
3
Wash hands with soap and water.
4
Wash hands with alcohol based hand sanitizer

Wash hands with soap and water.

The nurse assesses an elderly client with a diagnosis of dehydration and recognizes which finding as an early sign of dehydration?
1
Sunken eyes
2
Dry, flaky skin
3
Change in mental status
4
Decreased bowel sounds

Change in mental status

he nurse administers a pneumococcal vaccine to a 70-year-old client. The client asks "Will I have to get this every year like I do with the flu shot?" How should the nurse respond?
1
"You need to receive the pneumococcal vaccine every other year."
2
"The pneumococcal vaccine should be received in early autumn every year."
3
"You should get the flu and pneumococcal vaccines at your annual physical examination."
Correct 4
"It is unnecessary to have any follow-up injections of the pneumococcal vaccine after this dose.

It is unnecessary to have any follow-up injections of the pneumococcal vaccine after this dose

A nurse assesses for hypocalcemia in a postoperative client. One of the initial signs that might be present is:
1
Headache.
2
Pallor.
3
Paresthesias.
4
Blurred vision

Parasthesias

The nurse is caring for a client who is receiving therapy for vitamin B12 deficiency. Which finding indicates that the therapy is having the desired effect?
1
Normal serum electrolyte levels
2
Healthy skin integrity
3
Resolution of peripheral edema
4
Improved hemoglobin and hematocrit levels

Improved hemoglobin and hematocrit levels

What clinical finding does a nurse anticipate when admitting a client with an extracellular fluid volume excess?
1
Rapid, thready pulse
2
Distended jugular veins
3
Elevated hematocrit level
4
Increased serum sodium level

Distended jugular veins

A visitor from a room adjacent to a client asks the nurse what disease the client has. The nurse responds, "I cannot discuss any client’s illness with you." What legal issue supports the nurse’s response?
1
Libel
2
Slander
3
Negligence
4
Invasion of privacy

Invasion of privacy

A client has been admitted with a urinary tract infection. The nurse receives a urine culture and sensitivity report that reveals the client has Vancomycin Resistant Enterococcus (VRE). After notifying the physician, which action should the nurse take to decrease the risk of transmission to others?
1
Insert a urinary catheter.
2
Initiate Droplet Precautions.
3
Move the client to a private room.
4
Use a high efficiency particulate air (HEPA) respirator during care

Move the client to a private room

The nurse is monitoring a client’s hemoglobin level. The nurse recalls that the amount of hemoglobin in the blood has what effect on oxygenation status?
1
Except with rare blood disorders, hemoglobin seldom affects oxygenation status.
2
There are many other factors that impact oxygenation status more than hemoglobin does.
3
A low hemoglobin level causes reduced oxygen-carrying capacity.
4
Hemoglobin reflects the body’s clotting ability and may or may not impact oxygenation status.

A low hemoglobin level causes reduced oxygen-carrying capacity

A nurse addresses the needs of a client who is hyperventilating to prevent what complication?
1
Cardiac arrest
2
Carbonic acid deficit
3
Reduction in serum pH
4
Excess oxygen saturation

Carbonic acide deficit

A client with an abdominal wound infected with methicillin-resistant Staphylococcus aureus (MRSA) is scheduled for a computed tomography (CT) scan of the abdomen. To ensure client and visitor safety during transport, the nurse should implement which precaution?
1
No special precautions are required.
2
Cover the infected site with a dressing.
3
Drape the client with a covering labeled as biohazardous.
4
Place a surgical mask on the client.

Cover the infected site with a dressing

A nurse manager is evaluating the performance of the LPN/LVN who is supervising Unlicensed Assistive Personnel (UAP). What action indicates to the nurse manager that the LPN/LVN needs further instruction?
1
Requests that the UAP take vital signs on the clients assigned to their team.
2
Asks the UAP to assess the client’s response to a respiratory treatment
3
Instructs the UAP to communicate to a client that the meal trays will be delayed.
4
Collaborates with the UAP to determine the best time to ambulate a client.

Ask the UAP to assess the client’s response to a respiratory treatment

A nurse is taking care of a client who has severe back pain as a result of a work injury. What nursing considerations should be made when determining the client’s plan of care? (Select all that apply.)
1
Ask the client what is the client’s acceptable level of pain.
2
Eliminate all activities that precipitate the pain.
3
Administer the pain medications regularly around the clock.
4
Use a different pain scale each time to promote patient education.
5
Assess the client’s pain every 15 minutes

Ask the client what is the client’s acceptable level of pain. Administer the pain medications regularly around the clock.

When caring for a client with pneumonia, which nursing intervention is the highest priority?
1
Increase fluid intake.
2
Employ breathing exercises and controlled coughing.
3
Ambulate as much as possible.
4
Maintain an NPO status

Employ breathing exercises and controlled coughing.

A nurse is caring for a client for whom segmental postural drainage treatments are prescribed. The nurse should avoid scheduling the treatment at what time?
1
At bedtime
2
After a meal
3
One hour before a meal
4
One hour after awakening

After a meal

A child is being treated with oral ampicillin (Omnipen) for otitis media. What should be included in the discharge instructions that the nurse provides to the parents of the client?
1
Complete the entire course of antibiotic therapy.
2
Herbal fever remedies are highly discouraged.
3
Administer the medication with meals.
4
Stop the antibiotic therapy when the child no longer has a fever

Complete the entire course of antibiotic therapy.

A health care provider prescribes transdermal fentanyl (Duragesic) 25 mcg/hr every 72 hours. During the first 24 hours after starting the fentanyl, what is the most important nursing intervention?
1
Change the dose until pain is tolerable.
2
Manage pain with oral pain medication.
3
Assess the client for anticholinergic side effects.
4
Instruct the client to take the medication with food

Manage pain with oral pain medication

When suctioning a client with a tracheostomy, an important safety measure for the nurse is to:
1
Hyperventilate the client with room air prior to suctioning.
2
Apply suction only as the catheter is being withdrawn.
3
Insert the catheter until the cough reflex is stimulated.
4
Remove the inner cannula before inserting the suction catheter.

Apply suction only as the catheter is being withdrawn

A client has undergone a subtotal thyroidectomy. The client is being transferred from the post anesthesia care unit/recovery area to the inpatient nursing unit. What emergency equipment is most important for the nurse to have available for this client?
1
A defibrillator
2
An IV infusion pump
3
A tracheostomy tray
4
An electrocardiogram (ECG) monito

A tracheostomy tray

A nurse is obtaining a health history from the newly admitted client who has chronic pain in the knee. What should the nurse include in the pain assessment? (Select all that apply.)
1
Pain history including location, intensity and quality of pain
2
Client’s purposeful body movement in arranging the papers on the bedside table
3
Pain pattern including precipitating and alleviating factors
4
Vital signs such as increased blood pressure and heart rate
5
The client’s family statement about increases in pain with ambulation

Pain history including location, intensity and quality of pain Pain pattern including precipitating and alleviating factors

A client who sustained a large open wound as a result of an accident is receiving daily sterile dressing changes. To maintain sterility when changing the dressing, the nurse should:
1
Put the unopened sterile glove package carefully on the sterile field
2
Remove the sterile drape from its package by lifting it by the corners
3
Don sterile gloves before opening the package containing the field drape
4
Pour irrigation liquid from a height of at least three inches above the sterile containe

Remove the sterile drape from its package by lifting it by the corners.

The nurse assesses a client’s pulse and documents the strength of the pulse as 3+. The nurse understands that this indicates the pulse is:
1
diminished.
2
normal.
3
full.
4
bounding

full

A nurse manager is evaluating the performance of the LPN/LVN who is supervising Unlicensed Assistive Personnel (UAP). What action indicates to the nurse manager that the LPN/LVN needs further instruction?
1
Requests that the UAP take vital signs on the clients assigned to their team.
Correct 2
Asks the UAP to assess the client’s response to a respiratory treatment
3
Instructs the UAP to communicate to a client that the meal trays will be delayed.
4
Collaborates with the UAP to determine the best time to ambulate a client.

Asks the UAP to assess the client’s response to a respiratory treatment

A nurse is reviewing a client’s plan of care. What is the determining factor in the revision of the plan?
1
Time available for care
2
Validity of the problem
3
Method for providing care
4
Effectiveness of the interventions

Effectiveness of the interventions

A nurse has provided discharge instructions to a client who received a prescription for a walker to use for assistance with ambulation. The nurse determines that the teaching has been effective when the client:
1
Picks up the walker and carries it for short distances.
2
Uses the walker only when someone else is present.
3
Moves the walker no more than 12 inches in front of the client during use.
4
States that a walker will be purchased on the way home from the hospital.

Moves the walker no more than 12 inches in front of the client during use

A client with a terminal illness reaches the stage of acceptance. How can the nurse best help the client during this stage?
1
Acknowledge the client’s crying.
2
Encourage unrestricted family visits.
3
Explain details of the care being given.
4
Stay nearby without initiating conversation

Stay nearby without initiating conversation

A nurse is obtaining a health history from the newly admitted client who has chronic pain in the knee. What should the nurse include in the pain assessment? (Select all that apply.)
1
Pain history including location, intensity and quality of pain
2
Client’s purposeful body movement in arranging the papers on the bedside table
3
Pain pattern including precipitating and alleviating factors
4
Vital signs such as increased blood pressure and heart rate
5
The client’s family statement about increases in pain with ambulation

Pain history including location, intensity and quality of pain Pain pattern including precipitating and alleviating factors

The nurse should monitor for which involuntary physiological response in a client who is experiencing pain?
1
Crying
2
Splinting
3
Perspiring
4
Grimacing

Perspiring

What are the best ways for a nurse to be protected legally? (Select all that apply.)
1
Ensure that a therapeutic relationship with all clients has been established.
2
Provide care within the parameters of the state’s nurse practice act.
3
Carry at least $100,000 worth of liability insurance.
4
Document consistently and objectively.
5
Clearly document a client’s non-adherence to the medical regimen

Provide care within the parameters of the state’s nurse practice act. Document consistently and objectively. Clearly document a client’s non-adherence to the medical regimen

Which drug requires the nurse to monitor the client for signs of hyperkalemia?
1
Furosemide (Lasix)
2
Metolazone (Zaroxolyn)
3
Spironolactone (Aldactone)
4
Hydrochlorothiazide (HydroDIURIL)

Spironolactone

An 85-year-old client is alert and able to participate in care. The nurse understands that, according to Erikson, a person’s adjustment to the period of senescence will depend largely on adjustment to which developmental stage?
1
Industry versus inferiority
2
Identity versus role confusion
3
Generativity versus stagnation
4
Autonomy versus shame/doubt

Generativity versus role stagnation

A client is admitted to the hospital for an elective surgical procedure. The client tells a nurse about the emotional stress of recently disclosing being a homosexual to family and friends. What is the nurse’s first consideration when planning care?
1
Exploring the client’s emotional conflict
2
Identifying personal feelings toward this client
3
Planning to discuss this with the client’s family
4
Developing a rapport with the client’s health care provide

Identifying personal feelings toward this client

A dying client is coping with feelings regarding impending death. The nurse bases care on the theory of death and dying by Kübler-Ross. During which stage of grieving should the nurse primarily use nonverbal interventions?
1
Anger
2
Denial
3
Bargaining
4
Acceptance

Acceptance

A nurse applies an ice pack to a client’s leg for 20 minutes. The cold application will cause what physiological effect?
1
Local anesthesia
2
Peripheral vasodilation
3
Depression of vital signs
4
Decreased viscosity of blood

Local Anasthesia

While caring for a client with a Hemovac portable wound drainage system, the nurse observes that the collection container is half full. The nurse empties the container. What is the next nursing intervention?
1
Encircle the drainage on the dressing.
2
Irrigate the suction tube with sterile saline.
3
Clean the drainage port with an alcohol wipe.
4
Compress the container before closing the port

Compress the container before closing the port

When suctioning a client with a tracheostomy, an important safety measure for the nurse is to:
1
Hyperventilate the client with room air prior to suctioning.
2
Apply suction only as the catheter is being withdrawn.
3
Insert the catheter until the cough reflex is stimulated.
4
Remove the inner cannula before inserting the suction catheter.

Apply suction only as the catheter is being withdrawn.

What effect of povidone-iodine (Betadine) does a nurse consider when using it on the client’s skin before obtaining a specimen for a blood culture?
1
Makes the skin more supple
2
Avoids drying the skin as does alcohol
3
Eliminates surface bacteria that may contaminate the culture
4
Provides a cooling agent to diminish the feeling from the puncture wound

Eliminates surface bacteria that may contaminate the culture

A nurse is caring for a client who has a Hemovac portable wound suction device after abdominal surgery. What is the reason why the nurse empties the device when it is half full?
1
Emptying the unit is safer when it is half full.
2
Accurate measurement of drainage is facilitated.
3
Negative pressure in the unit lessens as fluid accumulates, interfering with further drainage.
4
Fluid collecting in the unit exerts positive pressure, forcing drainage back up the tubing and into the wound

Negative pressure in the unit lessens as fluid accumulates, interfering with further drainage.

The nurse instructs a client that, in addition to building bones and teeth, calcium is also important for:
1
Bile production.
2
Blood production.
3
Blood clotting.
4
Digestion of fats

Blood clotting

When meeting the unique preoperative teaching needs of an older adult, the nurse plans a teaching program based on the principle that learning:
1
Reduces general anxiety
2
Is negatively affected by aging
3
Requires continued reinforcement
4
Necessitates readiness of the learner

Requires continued reinforcement

A nurse is helping a client who observes the traditional Jewish dietary laws to prepare a dietary menu. What considerations should the nurse make?
1
Eating beef and veal is prohibited.
2
Consumption of fish with scales is forbidden.
3
Meat and milk at the same meal are forbidden.
4
Consuming alcohol, coffee, and tea are prohibited

Meat and milk at the same meal are forbidden

A client being treated for Influenza A (H1N1) is scheduled for a computed tomography (CT) scan. To ensure client and visitor safety during transport, the nurse should take which precaution?
1
Place a surgical mask on the client.
2
Other than Standard Precautions, no additional precautions are needed.
3
Minimize close physical contact.
4
Cover the client’s legs with a blanke

Place a surgical mask on the client

A nurse is reviewing a client’s plan of care. What is the determining factor in the revision of the plan?
1
Time available for care
2
Validity of the problem
3
Method for providing care
4
Effectiveness of the interventions

Effectiveness of the interventions

A nurse is evaluating the effectiveness of treatment for a client with excessive fluid volume. What clinical finding indicates that treatment has been successful?
1
Clear breath sounds
2
Positive pedal pulses
3
Normal potassium level
4
Increased urine specific gravity

Clear breath sounds

When providing care for a client with a nasogastric (NG) tube, the nurse should take measures to prevent what serious complication?
1
Skin breakdown
2
Aspiration pneumonia
3
Retention ileus
4
Profuse diarrhea

Aspiration pneumonia

A client who experienced extensive burns is receiving IV fluids to replace fluid loss. The nurse should monitor for which initial sign of fluid overload?
1
Crackles in the lungs
2
Decreased heart rate
3
Decreased blood pressure
4
Cyanosis

Crackles in the lungs

A client with a history of ulcerative colitis is admitted to the hospital because of severe rectal bleeding. The client engages in angry outbursts and places excessive demands on the staff. One day an unlicensed assistive personnel (UAP) tells the nurse, "I’ve had it. I am not putting up with that behavior. I’m not going in there again." What is the best response by the nurse?
1
"You need to try to be patient. The client is going through a lot right now."
2
"I’ll talk with the client. Maybe I can figure out the best way for us to handle this."
3
"Just ignore it and get on with your work. I’ll assign someone else to take a turn."
4
"The client’s frightened and taking it out on the staff. Let’s think of approaches we can take."

"The client’s frightened and taking it out on the staff. Let’s think of approaches we can take."

When nurses are conducting health assessment interviews with older clients, they should:
1
Leave a written questionnaire for clients to complete at their leisure
2
Ask family members rather than the client to supply the necessary information
3
Spend time in several short sessions to elicit more complete information from the clients
4
Keep referring to previous questions to ascertain that the information given by clients is correct

Spend time in several short sessions to elicit more complete information from the clients

A nursing supervisor sends unlicensed assistive personnel (UAP) to help relieve the burden of care on a short-staffed medical-surgical unit. Which tasks can be delegated to UAP? (Select all that apply.)
1
Taking routine vital signs.
2
Applying a sterile dressing.
3
Answering clients’ call lights.
4
Administering saline infusions.
5
Changing linens on an occupied bed.
6
Assessing client responses to ambulation.

Taking routine vital signs. Answering clients’ call lights. Changing linens on an occupied bed.

The nurse is interviewing a client admitted for uncontrolled diabetes after binging on alcohol for the past two weeks. The client states "I am worried about how I am going to pay my bills for my family while I am hospitalized." Which statement by the nurse would best elicit information from the client?
1
"You are worried about paying your bills?"
2
"Don’t worry; your bills will get paid eventually."
3
"When was the last time you were admitted for hyperglycemia?"
4
"You really shouldn’t be drinking alcohol because of your diagnosis of diabetes"

"You are worried about paying your bills?"

A nurse is caring for a newly admitted client in a long-term care facility. The nurse notes that the client has a decreased attention span and cannot concentrate. The nurse suspects which effects of sensory deprivation?
1
Cognitive response
2
Emotional response
3
Perceptual response
4
Physical response

Cognitive response

A nurse receives a shift report on four adult clients that are between the ages of 25-55. Which client should the nurse assess first?
1
Male client with a hemoglobin of 15.9
2
Female client on warfarin (Coumadin) with an International Normalized Ratio (INR) of 7.5
3
Female client taking daily calcium supplements with a serum calcium level of 9.4
4
Male client with a blood urea nitrogen (BUN) of 20 and a creatinine of 1.1

Female client on warfarin (Coumadin) with an International Normalized Ratio (INR) of 7.5

A nurse is discussing weight loss with an obese individual with Ménière’s disease. Which suggestion by the nurse is most important?
1
Limit intake to 900 calories a day.
2
Enroll in an exercise class.
3
Get involved in diversionary activities when there is an urge to eat.
4
Keep a diary of all foods eaten each day.

Keep a diary of all foods eaten each day

A client reports nausea, vomiting, and seeing a yellow light around objects. A diagnosis of hypokalemia is made. Upon a review of the client’s prescribed medication list, the nurse determines that what is the likely cause of the clinical findings?
1
Digoxin (Lanoxin)
2
Furosemide (Lasix)
3
Propranolol (Inderal)
4
Spironolactone (Aldactone

Digoxin

The nurse is caring for a client that underwent a rhinoplasty surgical procedure 5 hours ago. After administering pain medication, the nurse notes the client is swallowing frequently. The nurse understands that the cause of frequent swallowing is most likely caused from:
1
A normal response to the analgesic
2
Oral dryness caused by nasal packing
3
An adverse reaction to anesthesia
4
Bleeding posterior to the nasal packing

Bleeding posterior to the nasal packing

A client has seeds containing radium implanted in the pharyngeal area. What should the nurse include in the client’s plan of care?
1
Have the client void every two hours.
2
Maintain the client in an isolation room.
3
Spend time with the client to allow verbalization of feelings.
4
Wear two pairs of gloves when touching the client during care.

Maintain the client in an isolation room

A health care provider has prescribed isoniazid (Laniazid) for a client. Which instruction should the nurse give the client about this medication?
1
Prolonged use can cause dark concentrated urine.
2
The medication is best absorbed when taken on an empty stomach.
3
Take the medication with aluminum hydroxide to minimize GI upset.
4
Drinking alcohol daily can cause drug-induced hepatitis.

Drinking alcohol daily can cause drug-induced hepatitis

Which nursing interventions require a nurse to wear gloves? (Select all that apply.)
1
Giving a back rub
2
Cleaning a newborn immediately after delivery
3
Emptying a portable wound drainage
4
Interviewing a client in the emergency department
5
Obtaining the blood pressure of a client who is human immunodeficiency virus (HIV) positive.

Cleaning a newborn immediately after delivery Emptying a portable wound drainage

An older client who is receiving chemotherapy for cancer has severe nausea and vomiting and becomes dehydrated. The client is admitted to the hospital for rehydration therapy. Which interventions have specific gerontologic implications the nurse must consider? (Select all that apply.)
1
Assessment of skin turgor
2
Documentation of vital signs
3
Assessment of intake and output
4
Administration of antiemetic drugs
5
Replacement of fluid and electrolytes

Assessment of skin turgor Administration of antiemetic drugs Replacement of fluid and electrolytes

Health promotion efforts within the health care system should include efforts related to secondary prevention. Which activities reflect secondary prevention interventions in relation to health promotion? (Select all that apply.)
1
Encouraging regular dental checkups
2
Facilitating smoking cessation programs
3
Administering influenza vaccines to older adults
4
Teaching the procedure for breast self-examination
5
Referring clients with a chronic illness to a support group

Encouraging regular dental checkups Teaching the procedure for breast self-examination

To decrease abdominal distention following a client’s surgery, what actions should the nurse take? (Select all that apply.)
1
Encourage ambulation
2
Give sips of ginger ale
3
Auscultate bowel sounds
4
Provide a straw for drinking
5
Offer an opioid analgesic

Encourage ambulation Auscultate bowel sounds

A client becomes anxious after being scheduled for a colostomy. What is the most effective way for the nurse to help the client?
1
Administer the prescribed as needed (prn) sedative.
2
Encourage the client to express feelings.
3
Explain the postprocedure course of treatment.
4
Reassure the client that there are others with this problem

Encourage the client to express feelings.

Share This
Flashcard

More flashcards like this

NCLEX 10000 Integumentary Disorders

When assessing a client with partial-thickness burns over 60% of the body, which finding should the nurse report immediately? a) ...

Read more

NCLEX 300-NEURO

A client with amyotrophic lateral sclerosis (ALS) tells the nurse, "Sometimes I feel so frustrated. I can’t do anything without ...

Read more

NASM Flashcards

Which of the following is the process of getting oxygen from the environment to the tissues of the body? Diffusion ...

Read more

Unfinished tasks keep piling up?

Let us complete them for you. Quickly and professionally.

Check Price

Successful message
sending